You are on page 1of 65

SUBJECT-VERB PREVIEW: FINDING THE SUBJECT IN AN SAT QUESTION

Several questions on the SAT will test your ability to spot errors in subject-verb agreement. Checking for subject-verb agreement
seems like an easy task, but the subjects in SAT questions are often hidden by distracting words and phrases. Because you have only
35 minutes to answer all of the writing questions, you need to be able to match subjects and verbs quickly. Here are a few
guidelines.

START WITH THE VERB

The subject of a sentence is who or what the sentence is about. To find the subject, start with the verb. Ask yourself who or what is
doing the action of the verb. If the verb is not an action verb, but rather a linking verb or some other non-action verb, ask yourself
who or what is being described.
Kenny eats pancakes.
Who eats? Kenny eats. Kenny is the subject.
Omar is as talented as any contestant on American Idol.
Who is as talented as the Idol contestants? Omar is as talented as they are. Omar is the subject.
Kenny’s little brown dog has no spots.
Who or what has no spots? Kenny’s dog. The subject is dog, not Kenny. The word Kenny’s is a possessive that tells who
owns the dog.
Ignore possessives and adjectives that come before the noun.

Many of the subjects on the test are abstract nouns. Abstract nouns identify concepts, ideas, and other things that are
intangible.
Heidi’s creativity makes her a great artist.
What makes Heidi a great artist? Creativity. The subject is creativity. Again, ignore the possessive that comes before the
noun.
Failure is not an option.
What is not an option? Failure. Failure is the subject.

IGNORE PREPOSITIONAL PHRASES

The subject of a sentence is never in a prepositional phrase, so ignore prepositional phrases.


EXAMPLE A
A large wicker basket of oranges, pears, and apples sits in the center of the table.
The verb in this sentence is sits. What sits in the center of the table? A basket. The subject-verb pair is basket sits. “Of
oranges, pears, and apples” is a prepositional phrase. Remember, the subject of a sentence is never in a prepositional phrase.
TEST TIP: STRIKE OUT OR SET OFF THE PREPOSITIONAL PHRASES
Underlining, striking out (lightly), or putting parentheses around prepositional phrases and other descriptive phrases can help you
identify the subject-verb pair.
A large wicker basket of oranges, pears, and apples sits in the center of the table.
OR: A large wicker basket (of oranges, pears, and apples) sits in the center of the table.
EXAMPLE B
According to the latest report from the county’s top environmental engineer, the widespread use of pesticides on lawns,
trees, and shrubs has contaminated several local rivers and streams.
The verb form in this sentence is has contaminated. What has contaminated rivers and streams? The sentence tells us
that the widespread use of pesticides on lawns, trees, and shrubs has contaminated the rivers and streams. Which noun is the
subject? Remember that the subject is never in a prepositional phrase, so ignore the prepositional phrases “of pesticides”
and “on lawns, trees, and shrubs”:
the widespread use (of pesticides) (on lawns, trees, and shrubs) has contaminated local rivers and streams.
Striking the prepositional phrases reveals the subject, which is use. The subject-verb pair is use has contaminated. It is
the use of pesticides that has contaminated the rivers and streams.
IGNORE PHRASES SET OFF BY COMMAS

In checking SAT questions for subject-verb agreement, ignore any modifying phrases set off by commas. Examples are phrases that
start withaccompanied by, in addition to, as well as, together with, or coupled with. These phrases often make a sentence with one
subject look like a sentence with more than one subject, but in fact these phrases do nothing to change the subject of the sentence.
EXAMPLE A
Habitual tardiness, as well as poor attendance, is a cause for discipline.
What is the subject here? You might think there are two subjects: tardiness and attendance. But, in fact, there is only one
subject:tardiness. For purposes of identifying the subject of the sentence, ignore any modifying phrases between the noun
and the verb that are set off by commas. The subject-verb pair in this sentence is tardiness is.
Habitual tardiness, (as well as poor attendance), is a cause for discipline.
EXAMPLE B

Harold and Kumar’s new puppy, one of a litter of nine poodles, barks all night.

The verb in this sentence is barks. Who or what barks all night? The puppy. Ignore the phrase set off by commas.
Harold and Kumar’s new puppy, (one of a litter of nine poodles), barks all night.

THE SUBJECT AND VERB IN EACH CLAUSE MUST AGREE


Some of the sentences on the writing test have more than one clause, and each clause will have its own subject-verb pair.
Whereas their brother Darius is an accomplished athlete who works out every day, Sue and Marlene dislike sports and
spend most of their time watching reality television.
This sentence has several subject-verb pairings:
Darius is
who (athlete) works out
Sue and Marlene dislike and spend
Each of these subject-verb pairs must agree, which they do in this example.

SAMPLE SAT QUESTION


Try applying these rules to the following sample question:
Although whole grains and proteins are essential to good health, the abundance of vitamins, antioxidants, and fiber in fresh
fruits and vegetables, especially leafy greens and berries, make fresh produce the foundation of any healthful diet.
There are two clauses in this sentence, each with its own subject-verb pair:
(1) Although whole grains and proteins are essential to good health
(2) the abundance of vitamins, antioxidants, and fiber in fruits and vegetables, especially leafy greens and berries,  make fresh
produce the foundation of any healthful diet.
The subject and verb agree in the first clause: whole grains and proteins are essential. But what about the second clause? There are
many nouns in this sentence, but you can easily find the subject if you remember the rules. Ignore prepositional phrases and any
phrases set off by commas:
the abundance (of vitamins, antioxidants, and fiber) (in fruits and vegetables), (especially leafy greens and berries), make
fresh produce the foundation of any healthful diet.
Striking the distracting phrases exposes the error: abundance make. The subject abundance is singular and takes a singular verb:
Although whole grains and proteins are essential to good health, the abundance of vitamins, antioxidants, and fiber in
fruits and vegetables, especially leafy greens and berries, makes fresh produce the foundation of any healthful diet.
NOTE ABOUT YOU
The general rule is that every sentence must have a subject, and the subject cannot be implied. An exception to that rule, which
rarely appears on the test, is a sentence that expresses a command or request. In a sentence that directs someone to do something,
the subjectyou is understood.
Bring two pencils and a calculator to the test.
Do not call me on Thursday.
Go away!
CHAPTER 1
SUBJECT-VERB AGREEMENT

(1) GENERAL RULE

     Singular subjects take singular verbs. Plural subjects take plural verbs. Ignore distracting words and phrases.
Every high school student knows that singular subjects take singular verbs (e.g., a bird flies) and that plural subjects take plural
verbs (birds fly). Though this rule is simple, applying it to SAT questions is not. As you’ve already learned, the writers of SAT
questions often disguise errors in subject-verb agreement by inserting distracting words and phrases between the subject and the
verb. To isolate the subject and verb in an SAT question, ignore prepositional phrases, descriptive phrases, and phrases set off by
commas.

SINGULAR STILL SINGULAR

abundance abundance of vitamins and minerals

basket basket of apples, cherries, and pears

proposal proposal to impose license fees and import taxes

insistence insistence on rigorous workouts and strict diets

class class of thirty students

pesticide pesticide applied to lawns, gardens, and shrubs

principal principal, together with three teachers and a coach,

PLURAL STILL PLURAL

limits limits of power

petals petals of a single rose

dictionaries dictionaries, as well as a thesaurus,

TEST SAMPLE A
The International Red Cross announced today that a swarm of locusts have invaded southeastern Niger, where millions of
people are still suffering from food shortages caused by last year’s infestation.

EXPLAINED
This sentence follows a common pattern on the SAT. The singular subject swarm is followed by a prepositional phrase with a
plural object (of locusts). The prepositional phrase is a distraction. The subject swarm is singular and takes a singular verb.

REVISED
The International Red Cross announced today that a swarm of locusts has invaded southeastern Niger, where millions of
people are still suffering from food shortages caused by last year’s infestation.

TEST SAMPLE B
A shortage of dentists, as well as the inability to pay for dental care and high rates of tobacco use, have led to widespread
oral health problems in some of the state’s more rural counties.
EXPLAINED
The subject is shortage, which is singular. Ignore prepositional phrases and phrases set off by commas:
A shortage (of dentists), (as well as the inability to pay for dental care and high rates of tobacco use ), have led to
widespread oral health problems in some of the state’s more rural counties.
Striking these phrases exposes the error: shortage have led. Have led should be has led.

REVISED
A shortage of dentists, as well as the inability to pay for dental care and high rates of tobacco use,  has led to widespread oral
health problems in some of the state’s more rural counties.
TEST TIP: CHECK BOTH THE SUBJECT AND THE VERB IN THE IDENTIFYING-SENTENCE-ERRORS QUESTIONS
If a question in the identifying-sentence-errors section has an error in subject-verb agreement, either the subject or the verb will be
underlined. In almost all cases, the verb will be underlined, but occasionally the question writers will underline the subject instead of
the verb. Students sometimes miss the agreement error in these questions because the verb is not underlined. If you think you spot
an error in subject-verb agreement but the verb is not underlined, check to see if the subject is underlined.

This sentence has an error in subject-verb agreement. The singular noun cause requires the singular verb lies: the cause lies. The
plural verb lie is not underlined, however, so the only way to correct the subject-verb error in this sentence is to change the singular
noun causeto the plural noun causes. The answer is B.

(2) MULTIPLE SUBJECTS

     Two or more subjects joined by and take plural verbs. Exceptions are multiple subjects preceded
by each or every, and multiple subjects that form a single unit (e.g., peanut butter and jelly).
Two or more subjects joined by and take plural verbs, regardless of whether the subjects are all singular, all plural, or a mix of
singular and plural.
Lucas, Margene, and Omar are working as lifeguards this summer.
Two soccer fields and a tennis court are welcome additions to the park.
Exceptions are subjects preceded by each or every, and multiple subjects that form a single unit.
Each lion and tiger has its own cage at the zoo.
Every student and teacher carries an identification card.
Peanut butter and jelly is my favorite sandwich.

TEST SAMPLE
Every Thanksgiving Diego bakes a pecan pie, and his cousin Yvonne makes a green bean casserole, but the stuffing, roasting,
and carving of the turkey is always left to Diego’s father, who is a respected chef and cookbook author.

EXPLAINED
Stuffing, roasting, and carving are three subjects joined by and. The verb should be plural.

REVISED
Every Thanksgiving Diego bakes a pecan pie, and his cousin Yvonne makes a green bean casserole, but the stuffing, roasting,
and carving of the turkey are always left to Diego’s father, who is a respected chef and cookbook author.
A NOTE ABOUT GERUNDS
Words such as stuffing, roasting, and carving are called gerunds. Gerunds are forms of verbs ending in -ing that act as nouns. A
gerund is singular and takes a singular verb (e.g., skiing is fun). Two or more gerunds joined by and take a plural verb (e.g.,
skiing and biking arefun). Questions on the SAT sometimes have gerunds or gerund phrases as subjects.
In the United States, extracting gold and silver from their ores is often accomplished using the cyanide extraction process.
Question writers sometimes try to make multiple subjects seem like one unit by placing a single adjective before both subjects
(e.g., flavored tea and coffee, warm air and water). Don’t fall for that trick. There are very few subjects joined by and that are
treated as singular, and the SAT is generally not interested in testing your ability to spot them. Don’t expect to find the
singular macaroni and cheese or peanut butter and jelly as subjects in SAT questions. Question writers are much more interested in
testing your knowledge of the rule that subjects joined by andare plural:
Scented soap and bath oil are popular Mother’s Day gifts.
Warm sun and sand make the beach the perfect place to spend a summer day.

(3) INVERTED (BACKWARD) SENTENCES

     In inverted (backward) sentences, the verb must agree with the subject or subjects that follow it.
Some of the sentences on the test are constructed so that the subject follows the verb. Often these sentences begin with a
prepositional phrase (e.g., Behind the door stands Boo Radley). Don’t let the placement of the verb confuse you. If the verb appears
first, look at the subject or subjects that follow. If the verb is followed by a plural subject or by multiple subjects, the verb must be
plural. Consider the following example:
In the tool kit was a screwdriver and a wrench.
What was in the toolkit? A screwdriver and a wrench. The verb must be plural: In the tool kit were a screwdriver and a wrench.

TEST SAMPLE
Also passed by Congress was an energy bill that establishes a cap and trade program and an economic stimulus bill that
boosts investment in alternative energy.

EXPLAINED
This sentence has two subjects: an energy bill and an economic stimulus bill. Multiple subjects require a plural verb. The error
in this sentence is the singular verb was.

REVISED
Also passed by Congress were an energy bill that establishes a cap and trade program and an economic stimulus bill that
boosts investment in alternative energy.
NOTE ABOUT INVERTED SENTENCES
Not every sentence that can be reversed is an inverted sentence. A sentence such as My favorite meal is roast beef and mashed
potatoesis not a backward sentence. Rather, it is a sentence that connects a singular subject with multiple nouns. In these types of
sentences, unlike backward sentences, the subject comes before the verb.
The treatment is rest and massage therapy. [subject = treatment]
A common cold remedy is chicken soup and tea with lemon. [subject = remedy]
Her favorite breakfast is scrambled eggs and wheat toast. [subject = breakfast]
In inverted sentences, there may be a noun before the verb, but that noun is not the subject of the sentence because that noun is
not what the sentence is about. In inverted sentences, the noun or pronoun that follows the verb is the subject.
Along the northeast coast of Australia lies the world’s largest coral reef. [subject = reef]
Available in the principal’s office are attendance logs and disciplinary records. [subjects = logs + records]

(4) THERE IS AND THERE ARE

   There is should be used if the subject that follows is singular. There are should be used if the subject that follows is
plural.
This rule is similar to the rule for inverted sentences. In both cases, the subject follows the verb. If the subject is singular, use  there
is. If the subject is plural, use there are.
Though Spiffy’s specializes in vegetarian cooking, there are several delicious meat entrees on the menu.
The sales clerk reminded us that there is an update available for this software.

TEST SAMPLE
Television broadcasts were scheduled to shift from analog to digital last February, but federal officials postponed the
transition after cable companies reported that there were a shortage of digital converter boxes available for purchase by
consumers.

EXPLAINED
Shortage is singular, so there were should be there was.

REVISED
Television broadcasts were scheduled to shift from analog to digital last February, but federal officials postponed the
transition after cable companies reported that there was a shortage of digital converter boxes available for purchase by
consumers.

(5) COLLECTIVE NOUNS

     Collective nouns such as class, committee, team, and council are singular and take singular verbs.
Collective nouns identify groups of things or people that are treated as a single unit. Examples are  class, committee, team, group,
union, andcouncil. Although grammar experts agree that collective nouns sometimes take plural verbs, the collective nouns on the
SAT are always treated as singular and take singular verbs and pronouns.

TEST SAMPLE A
The restaurant’s chef is extraordinarily talented and his food is terrific, but the restaurant’s staff of aspiring actors and
models treat the customers like unwelcome house guests.

EXPLAINED
Staff is a collective noun that takes a singular verb. Ignore the prepositional phrase of aspiring actors and models.

REVISED
The restaurant’s chef is extraordinarily talented and his food is terrific, but the restaurant’s  staff of aspiring actors and
models treats the customers like unwelcome house guests.

TEST SAMPLE B
The prosecutor had argued that the videotape shows the defendant robbing the bagel shop, but the jury of eight men and
four women were unimpressed with the grainy and distorted footage and found the defendant not guilty.

EXPLAINED
Jury is a collective noun that takes a singular verb. Ignore the prepositional phrase of eight men and four women.

REVISED
The prosecutor had argued that the videotape shows the defendant robbing the bagel shop, but the jury of eight men and
four womenwas unimpressed with the grainy and distorted footage and found the defendant not guilty.
Common collective nouns include:
agency family
audience firm
band group
board jury
class orchestra
club panel
collection partnership
committee public
community society
company staff
council team
crowd union
faculty  
(6) OR AND NOR

   When two subjects are separated by or or nor, the verb must agree with the subject that is closer to the verb.
Many students have trouble with this rule because applying it often results in sentences that “sound wrong” but are grammatically
correct:Neither my sisters nor my cousin is going to the poetry reading tonight. That sentence may sound wrong, but it is
grammatically correct. Don’t rely on your ear to guide you on the test. Know the rules of grammar and apply them.

TEST SAMPLE
Robert often tells us that he earned a degree in political science from Harvard in 2005, but neither the dean of academic
affairs nor the professors in the political science department remembers him.

EXPLAINED
This sentence has the conjunction pair neither . . . nor. The two subjects separated by nor are dean and professors.
Professors, a plural noun, is closer to the verb than dean, so the verb must be plural. NOTE: Did the prepositional phrases
confuse you? Remember that striking or putting parentheses around prepositional phrases can help you identify subjects.
Robert often tells us that he earned a degree in political science from Harvard in 2005,  but neither the dean (of academic
affairs)nor the professors (in the political science department) remembers him.
Again, striking out the intervening phrases exposes the error. Remembers should be remember.

REVISED
Robert often tells us that he earned a degree in political science from Harvard in 2005, but  neither the dean of academic
affairs northe professors in the political science department remember him.
If we change the order of the two subjects, the verb must change as well:
Robert often tells us that he earned a degree in political science from Harvard in 2005, but neither the professors in the
political science department nor the dean of academic affairs remembers him.
NOTE ABOUT NEITHER AND EITHER
When neither or either is used alone, without nor or or, it always takes a singular verb and a singular pronoun.
Neither proposal is acceptable.
Neither dog is a good pet.

(7) INDEFINITE PRONOUNS

   Indefinite pronouns such as everybody, anyone, someone, and nothing are singular and take singular verbs. Ignore
intervening phrases.
Indefinite pronouns are generic pronouns. They include words such as everybody, everyone, everything, anybody, anyone, anything,
somebody, someone, something, nobody, no one, and nothing. Indefinite pronouns take singular verbs.

TEST SAMPLE
Everybody except the very young, the elderly, and the chronically ill have been placed on a waiting list for the flu shot.

EXPLAINED
The subject of the sentence is everybody. Everybody is an indefinite pronoun that requires a singular verb. The phrase except
the very young, the elderly, and the chronically ill is there to distract you. Ignore the intervening phrase.

REVISED
Everybody except the very young, the elderly, and the chronically ill has been placed on a waiting list for the flu shot.

(8) THE NUMBER OF AND THE PERCENTAGE OF

   The number of and the percentage of always take singular verbs.


TEST SAMPLE A
Although a typical trip to Antarctica costs more than $9,000 and the conditions on the Antarctic continent are harsh and
turbulent, the number of visitors to Antarctica have quadrupled in the last decade.

EXPLAINED
The number of always takes a singular verb. Have quadrupled should be has quadrupled.

REVISED
Although a typical trip to Antarctica costs more than $9,000 and the conditions on the Antarctic continent are harsh and
turbulent, the number of visitors to Antarctica has quadrupled in the last decade.

TEST SAMPLE B
The percentage of teenagers who undergo plastic surgery procedures such as chin implants, nose jobs, and ear reshaping
have increased every year since 1995.

EXPLAINED
The percentage of takes a singular verb. The plural nouns that follow the percentage of are there to distract you.

REVISED
The percentage of teenagers who undergo plastic surgery procedures such as chin implants, nose jobs, and ear
reshaping has increased every year since 1995.
NOTE ABOUT “A NUMBER OF”
A number of is different from the number of. A number of takes a plural verb.
A number of former Wall Street brokers are now working at Quickie Mart.
A number of celebrities have written children’s books.

(9) ALL, SOME, AND MOST

 All, some, and most may be singular or plural, depending on what follows them.


When all, some, or most is followed by a noun, the verb should match the noun: A singular noun takes a singular verb (e.g.,  all flour
is milled),and a plural noun takes a plural verb (e.g., all kangaroos hop). When all, some, or most is followed by a prepositional
phrase, the verb should match the object of the preposition. If the object of the prepositional phrase is plural, the verb should be
plural (e.g., most of the cookies were eaten). If the object of the prepositional phrase is singular, the verb should be singular
(e.g., most of the cookie was eaten). Consider the following examples:
All of the supporting performances, including those of the ensemble, were outstanding.
All the milk in Jim’s refrigerator is sour.
Some of the world’s most beautiful paintings are in the Louvre.
All of the cheese was put on a platter.
Most of the unemployment is in the banking sector.
Some water is too contaminated to use for cooking.
A NOTE ABOUT NONE
Grammar experts generally agree that none can be either singular or plural depending on its intended meaning. Questions on the
SAT, however, always treat none as singular.
None of the news anchors on the major television networks is a union member.
None of the bowling championship was televised.

(10) PORTIONS, PARTS, AND FRACTIONS

 Subjects that represent portions, parts, or fractions may be singular or plural, depending on what follows them.
Subjects that begin with phrases such as the majority of, a majority of, a minority of, the minority of, and a percentage of (but
NOT thepercentage of) may be singular or plural, depending on what follows them. The same is true of subjects that begin with
fractions or percentages, such as forty-four percent of or one-fifth of. These types of subjects take a singular verb if the object of the
preposition is singular but a plural verb if the object of the preposition is plural.
A majority of history majors want to attend law school after graduation.
A minority of the electorate favors an increase in property taxes.
A small percentage of union members oppose the new contract.
Thirty-one percent of high school students have part-time jobs.
Two-thirds of American voters favor an overhaul of the banking system.
Two-thirds of the American electorate favors an overhaul of the banking system.
Half of the houses in Cooper City have basements.

(11) DATA, CRITERIA, AND MEDIA

 Data, criteria, and media are plural and take plural verbs.


Data, criteria, and media are the plural forms of datum, criterion, and medium, respectively, and therefore, they are plural. Many
writers treat these words as singular, but the traditional rule provides that they should be treated as plural. Though  data,
criteria, and media are the most likely to be tested, a number of other words cause confusion:

SINGULAR PLURAL

addendum addenda

alumnus alumni

bacterium bacteria

curriculum curricula

memorandum memoranda

phenomenon phenomena

TEST SAMPLE
The percentage of applicants accepted to colleges in the Ivy League has decreased over the last decade, even though the
criteria for admission to these colleges has not changed.

EXPLAINED
Criteria is plural and takes a plural verb.

REVISED
The percentage of applicants accepted to colleges in the Ivy League has decreased over the last decade, even though
the criteria for admission to these colleges have not changed.

(12) WHO, WHICH, AND THAT

     Who, which, and that are singular when they refer to singular nouns but plural when they refer to plural
nouns.
Who, which, and that are used to introduce clauses that describe a noun or pronoun in the sentence. These clauses are called
relative clauses.Who, which, and that take plural verbs when they refer to plural nouns. They take singular verbs when they refer to
singular nouns.
EXAMPLES (relative clauses italicized):
People who live in the tropics are accustomed to high humidity.
[Who refers to the plural noun people.]
Lamar’s cousin works with two scientists who were nominated for a Nobel Prize.
[Who refers to the plural noun scientists.]
Lauren and Josh frequently travel to Amsterdam, which is their favorite city.
[Which refers to the singular noun Amsterdam.]
Dexter always orders vegetables that have been steamed or boiled.
[That refers to the plural noun vegetables.]
Note that relative clauses have verbs, but these verbs are not the main verbs in the sentence. If you strike the relative clause, the
main clause remains.
Sales clerks who work at Quickie Mart get a discount on hot dogs.

TEST SAMPLE
The Food and Drug Administration has identified Yummy Tummy Vegetable Company, based in Lubbock, Texas, as the
source of the spinach, onions, and lettuce that was contaminated with E. coli bacteria.

EXPLAINED
What does that refer to? That refers to spinach, onions, and lettuce. Multiple subjects connected by and require a plural
verb: the spinach, onions, and lettuce were contaminated.

REVISED
The Food and Drug Administration has identified Yummy Tummy Vegetable Company, based in Lubbock, Texas, as the source
of the spinach, onions, and lettuce that were contaminated with E. coli bacteria.
NOTE ABOUT MATCHING WHO, THAT, AND WHICH TO NOUNS
Finding the noun that who, which, or that refers to is different from finding the subject of the sentence. Sometimes
who, which, or thatrefers to the noun that is the subject of the sentence (e.g., The band that plays at Charlie’s is terrible), but often it
does not.
The box of antique dolls that were once owned by Queen Victoria has been sent to the curator of the Royal Museum.
That refers to dolls, which is not the subject of the sentence. The subject of the sentence is box, which is singular and is paired with
the singular verb has: The box . . . has been sent.

(13) ONE OF THOSE WHO VS. THE ONLY ONE OF THOSE WHO

 One of those who takes a plural verb. The only one of those who takes a singular verb.
Review the following sentences, and ask yourself which is correct.
(A) Angelina Jolie is one of those actresses who has international appeal.
(B) Angelina Jolie is one of those actresses who have international appeal.
The correct answer here is (B): Angelina Jolie is one of those actresses who have international appeal. This sentence places Angelina
Jolie in a particular category of people who share a trait: actresses who have international appeal. The who in the sentence refers to
the actresses, not to Angelina. Actresses is plural, so the plural verb have must be used. In this sentence, Angelina is just one of the
many actresses who have international appeal.
Now look at these two choices:
(A) Angelina Jolie is the only one of those actresses who has international appeal.
(B) Angelina Jolie is the only one of those actresses who have international appeal.
The correct answer here is (A). What’s the difference? In this sentence, Angelina is not being placed in a category. To the contrary,
she is being singled out for having a quality the other actresses do not have. The who in the sentence is one (Angelina), not the
actresses. Angelina is the only one who has international appeal.
Here are some other examples:
Saw is one of those movies that make audiences scream.
Dr. Markoff is one of those professors who enjoy meeting with students.
Stephen King is the only one of those authors who has won an Emmy.
Professor Spock is one of the scientists who have raised concerns about cloning.
Omar is the only one of the American Idol contestants who has perfect pitch.

TEST SAMPLE
The principal selected Mr. Garza as the chaperone for the class trip to South America because he is the only one of the
Spanish teachers who speak both Spanish and Portuguese fluently.

EXPLAINED
In this sentence who refers to one, which is singular. Mr. Garza is the only one who speaks both Spanish and Portuguese
fluently.

REVISED
The principal selected Mr. Garza as the chaperone for the class trip to South America because he is the  only one of the
Spanish teachers who speaks both Spanish and Portuguese fluently.

(14) TITLES

 Titles of books, plays, and movies take singular verbs.


Titles take singular verbs, even if the titles themselves are plural. This rule is unlikely to be tested in the multiple-choice questions,
but you should know the rule for the essay. If you refer to a book, poem, or other title in your essay, make sure you use a singular
verb.
The Chronicles of Narnia is a series of fantasy novels written by C.S. Lewis.
The Grapes of Wrath describes the hardships of the Joad family during the Great Depression.

REVIEW QUESTIONS: SUBJECT-VERB AGREEMENT

1. Although students and parents [opposes] [oppose] the new attendance policy, the faculty [supports] [support] it.
2. The mayor of Buffalo, along with the governor of New York and two state senators, [has] [have] asked Congress for funds to
clean up the oil spill in Lake Erie.
3. A number of dairy farms and meat packing plants [has] [have] issued recalls for products that may contain a dangerous
strain ofE. coli bacteria.
4. Each manager, assistant manager, and supervisor [receives] [receive] training in customer service.
5. Biologists are concerned that the number of spawning grounds for salmon and trout [is] [are] declining.
6. A box of pencils, together with two pens, [is] [are] on the desk.
7. Available from the park ranger [is] [are] a map of the trail and a pamphlet about hiking safety.
8. Each of the juveniles who [was] [were] found guilty of vandalizing Chuck E. Cheese [was] [were] ordered to attend anger
management therapy, perform twenty hours of community service, and write a letter of apology to the store manager.
9. Taking four Advanced Placement classes [is] [are] challenging but manageable.
10. Everyone but Kendra, Heidi, and Spencer [has] [have] adapted to the heat and humidity of the Costa Rican jungle.
11. Neither Harvey nor the Garcia twins [was] [were] invited to the picnic.
12. Professor Penn is one of the scientists who [has] [have] raised concerns about climate change.
13. The alumni of the university [is] [are] angry about the football team’s antics off the field.
14. The coach’s insistence on rigorous workouts and strict diets [has] [have] caused many players to quit the team.
15. The Spanish class, together with the Fine Art class, [is] [are] going on a field trip to the Goya exhibit at the art museum.

CHAPTER 2
PRONOUNS

(1) GENERAL RULE

     Pronouns in the subjective case (I, you, he, she, it, we, they) should be used for subjects. Pronouns in the
objective case(me, you, him, her, it, us, them) should be used for objects.
The subject is the person or thing that does the action of the verb. If the subject is a pronoun, the pronoun must be in the subjective
case.
He went to the movies, but I went home.
She rescues cats.
Are they selling the car?
An object receives the action of the verb. If the object is a pronoun, the pronoun must be in the objective case:
Omar called me.
Harlan and Kim visited us last Thursday.
I asked Carrie to call him and her.
Marjorie called them last Thursday.
TEST TIP: BREAK SENTENCES INTO MINI-SENTENCES.
Some sentences have more than one clause and more than one pronoun. The best way to identify the role of a pronoun in a
complex sentence is to break the sentence into mini-sentences and look at the role that the pronoun plays in its mini-sentence. Look
for mini-sentences that have at least one subject and a verb. Some mini-sentences will have a subject, a verb, and an object.
Professor Lee told me that Marcus and me are the leading candidates for the economics prize.
There are two mini-sentences:
Professor Lee told me
Marcus and me are the leading candidates (for the economics prize)
You can see that the first mini-sentence is correct, but the second has a pronoun error. The subjects should be  Marcus and
I, not Marcus and me: Marcus and I are the leading candidates.

(2) MULTIPLE SUBJECTS AND OBJECTS

     In clauses with more than one subject or object, test the pronoun on its own.
Identifying the correct pronoun case is fairly easy in sentences with only one subject or one object. It is much more challenging,
however, to pick the right pronoun when the pronoun is paired with a noun or another pronoun. To determine whether a pronoun is
in the correct case, test it on its own.

TEST SAMPLE A
After Ariana fell on stage, fracturing her big toe, her and the other principal dancers informed the artistic director that they
would not perform again until the ballet company fixed the rickety stage and paid Ariana’s hospital bill.

EXPLAINED
To determine whether her is the correct pronoun, test it on its own: Her informed the artistic director. Her is incorrect. The
pronoun should be she. She and the other principal dancers are the subjects of the sentence.

REVISED
After Ariana fell on stage, fracturing her big toe, she and the other principal dancers informed the artistic director that they
would not perform again until the ballet company fixed the rickety stage and paid Ariana’s hospital bill.

TEST SAMPLE B
My sister Louise and me will attend the University of Washington in the fall: Louise plans to major in computer science and
pursue a career as a programmer, while I intend to study theater and become an actress.
EXPLAINED
Me is incorrectly used as a subject in this sentence. The objective pronoun me should be the subjective pronoun I. If you had
trouble spotting the error, test the pronoun on its own: Me will attend the University of Washington. The error is plain.

REVISED
My sister Louise and I will attend the University of Washington in the fall: Louise plans to major in computer science and
pursue a career as a programmer, while I intend to study theater and become an actress.

TEST SAMPLE C
Impressed with our previous studies of Arctic wildlife, the National Science Foundation awarded Cora and I a grant to travel
to the Arctic region to study polar bear behavior.

EXPLAINED
I is incorrectly used as an object in this sentence. The subject pronoun I should be changed to the object pronoun me. Try
dropping Corato test the pronoun on its own: The National Science Foundation awarded I a grant. Striking out Cora exposes
the error. I should be me: The National Science Foundation awarded me a grant. Cora and me are the objects of the action:
the grant was awarded to them.

REVISED
Impressed with our previous studies of Arctic wildlife, the National Science Foundation awarded Cora and me a grant to
travel to the Arctic region to study polar bear behavior.

(3) OBJECTS OF PREPOSITIONS

     Pronouns that are the objects of prepositions should be in the objective case.
Prepositional phrases end in objects. Pronouns that are objects of prepositions should be in the objective case:
from them
to us
between him and us
with her and him
by me
Pay particular attention to prepositional phrases beginning with between. Although people frequently say, “just between you and I,”
the correct prepositional phrase is “between you and me.”

TEST SAMPLE
The frequent disagreements between my father and I have ruined many family dinners and affected my relationship with
my mother, who leaves the room every time my father and I argue.

EXPLAINED
Between my father and I is a prepositional phrase. The pronouns in a prepositional phrase should be in the objective case.
The subjective case I should be changed to the objective case me.

REVISED
The frequent disagreements between my father and me have ruined many family dinners and affected my relationship with
my mother, who leaves the room every time my father and I argue.

(4) PRONOUNS AFTER THAN AND AS

 Use than I or as I when shortened from than I am, than I did, than I have, as I am, as I did, or as I have. Apply this rule
to he, she, they, and we as well.
Which sentence is correct?
(1) Mark Twain is a better writer than me.
(2) Mark Twain is a better writer than I.
Most people think that the first sentence is correct because it “sounds right.” The correct sentence, however, is sentence (2): Mark
Twain is a better writer than I. The sentence Mark Twain is a better writer than I is a shortened version of the sentence Mark Twain
is a better writer than I am. To determine which pronoun to use after than or as, add the word or words that are omitted from the
sentence. Remember that although the example uses the pronoun I, this rule also applies to the other subjective pronouns (he, she,
they, we).

WRONG Omar is a much better tennis player than me.

RIGHT Omar is a much better tennis player than I [am].

WRONG I hope you are as happy as her when you go to college.

RIGHT I hope you are as happy as she [is] when you go to college.

WRONG I baked more cupcakes than him.

RIGHT I baked more cupcakes than he [did].

TEST SAMPLE
Although the judges have not yet announced the winners of the talent contest, Spiro and I expect to lose because the
singing jugglers put on a much better show than us.

EXPLAINED
Is us the correct pronoun to use here? Add the words that finish the thought: the singing jugglers put on a better show than
us [did].Rephrasing the sentence reveals the error. Us should be we: the singing jugglers put on a better show than we [did].

REVISED
Although the judges have not yet announced the winners of the talent contest, Spiro and I expect to lose because the singing
jugglers put on a much better show than we.
NOTE ABOUT THAN + PRONOUN
Sometimes than+ objective pronoun (e.g., than me, than us) accurately expresses the writer’s intent. Consider the following
sentence: I like Bill more than him. That sentence is accurate if the writer means that he likes Bill more than he likes someone else: I
like Bill more than [I like] him. The sentence is incorrect if the writer means that he likes Bill more than someone else likes Bill: I like
Bill more than he [does]. In either case, the words that have been omitted should be added to the sentence so that the meaning is
clear.
If than+ objective pronoun is tested on the SAT, the error must be clear. As a result, the error would likely follow the pattern  Mark
Twain is a better writer than me, which can be interpreted only as Mark Twain is a better writer than I [am]. Because this rule is
confusing and difficult to test, it is falling out of favor as a subject for testing.

(5) REFLEXIVE PRONOUNS: MYSELF, HERSELF, HIMSELF, AND ITSELF

   Reflexive pronouns such as myself, herself, and himself are not proper substitutes for subjective or objective
pronouns. Use reflexive pronouns only for emphasis or when the subject and the object are the same person.
Myself, herself, himself, yourself, itself, ourselves, and themselves are reflexive pronouns. Reflexive pronouns are not substitutes for
subjective and objective pronouns. Rather, they are a special type of pronoun used in two circumstances. The first is for emphasis:
Steven Spielberg himself approved the script.
The players themselves were the most disappointed by the loss.
The second is in sentences in which the subject and the object are the same person:
He hurt himself playing soccer.
I looked at myself in the mirror.
Julia went to the mall by herself.
TEST SAMPLE
The coach asked Caleb and myself to organize the equipment closet, clean the locker room, and take the team’s uniforms to
the laundry room.

EXPLAINED
Neither reason for using the reflexive pronoun myself applies here. Myself should be me. If you are unsure
whether me or I should be used, test the pronoun on its own:
The coach asked me to organize
The coach asked I to organize
The correct pronoun is me. Caleb and me are the objects in this sentence.

REVISED
The coach asked Caleb and me to organize the equipment closet, clean the locker room, and take the team’s uniforms to the
laundry room.

(6) AMBIGUOUS PRONOUNS

     A pronoun must clearly refer to a specific noun in the sentence. If a pronoun could refer to more than one
noun in the sentence, the pronoun reference is ambiguous.
Every pronoun must have an explicit and clear antecedent, which means that a pronoun must refer to a specific noun (or some other
pronoun) in the sentence. If a pronoun that could refer to more than one noun in the sentence, the pronoun reference is
ambiguous. A mistake that many students make is to overlook the ambiguous pronoun because that pronoun is more logically
connected to one noun in the sentence than to any of the others. Do not fall for that trick on the test.

TEST SAMPLE A
When the police officers brought the suspects to the police station, the detective questioned them thoroughly.

EXPLAINED
The pronoun them could refer to the officers or the suspects, or both the officers and the suspects. You might assume that
the pronounthem refers to the suspects because that choice is more logical than the others, but if you have to guess which
noun the pronoun refers to, the pronoun reference is ambiguous. Don’t make assumptions when reviewing test questions.

REVISED
When the police officers brought the suspects to the police station, the detective questioned the suspects thoroughly.

TEST SAMPLE B
After arguing for two years about the unsightly pile of scrap metal on Bill’s front lawn, Bill and his neighbor finally agreed
that he should clean up the mess and take the scrap metal to the junkyard.

EXPLAINED
The pronoun he is ambiguous. Does it refer to Bill or his neighbor? Don’t assume that he refers to Bill just because that choice
seems more logical.

REVISED
After arguing for two years about the unsightly pile of scrap metal on Bill’s front lawn, Bill and his neighbor finally agreed
that Billshould clean up the mess and take the scrap metal to the junkyard.

TEST SAMPLE C
Simon’s lawyer advised Janice that she needs to be in the courtroom when the jury announces its verdict.

EXPLAINED
The pronoun she is ambiguous. Does it refer to Simon’s lawyer or to Janice? Beware of sentences that refer to someone by
position and leave open the question of gender (e.g., Sam’s architect, her personal assistant, Sheila’s doctor).
REVISED
Simon’s lawyer advised Janice that Janice needs to be in the courtroom when the jury announces its verdict.

(7) PRONOUN AGREEMENT: SINGULAR OR PLURAL?

     A pronoun that refers to a singular noun must be singular. A pronoun that refers to a plural noun must be
plural.
This rule is always tested. Singular nouns that identify people without specifying gender, such as teacher, coach,
physician, or scientist,require the singular pronoun construction he or she. Singular nouns that identify things, such as store, beauty,
rose, or instruction, require the singular pronoun it. Plural nouns that identify people or things such as animals, specialists,
scientists, and shovels, as well as multiple subjects such as the iguana and the rattlesnake, Jon and Kate, and Harold and
Kumar, require the plural they.

TEST SAMPLE A
According to the school’s honor code, a student accused of cheating or plagiarism may admit wrongdoing and accept
suspension, or they can request a hearing to rebut the charge.

EXPLAINED
The sentence uses the plural they to refer to the singular student. Student is singular and requires the singular pronoun
construction he or she.

REVISED
According to the school’s honor code, a student accused of cheating or plagiarism may admit wrongdoing and accept
suspension, or he or she can request a hearing to rebut the charge.

TEST SAMPLE B
Unlike film producers, who must promote his or her films by appearing at premieres and film festivals, Broadway producers
hardly ever participate in promotional events.

EXPLAINED
The error in this sentence is the singular pronoun construction his or her. Who does his or her refer to? His or her refers
to film producers, which is plural. The pronoun should also be plural.

REVISED
Unlike film producers, who must promote their films by appearing at premieres and film festivals, Broadway producers hardly
ever participate in promotional events.

(8) PRONOUN AGREEMENT: COLLECTIVE NOUNS

     Collective nouns such as team, class, union, committee, and council take singular pronouns.


This rule is frequently tested on the SAT. When you spot a collective noun such as class, committee, panel, team, or union in a
sentence, make sure the pronoun that refers to the noun is it, not they.

TEST SAMPLE A
Margene wanted to join the army after graduation, but they rejected her offer to enlist because she has flat feet and a
sleeping disorder.

EXPLAINED
Army is a collective noun that takes a singular pronoun. The pronoun they is plural. The singular pronoun it should be used.

REVISED
Margene wanted to join the army after graduation, but it rejected her offer to enlist because she has flat feet and a sleeping
disorder.
TEST SAMPLE B
The mayor was elected on her promise to bring high-speed rail to the city, but the city council is reluctant to give their
approval to such an expensive project.

EXPLAINED
Council is a collective noun that takes a singular pronoun.

REVISED
The mayor was elected on her promise to bring high-speed rail to the city, but the city council is reluctant to give  its approval
to such an expensive project.
TEST TIP: THEY VS. IT
Always be on alert for the pronouns they, their, and them. In everyday speech, people use they, their, and them when referring not
only to collective nouns such as team or class but also to all kinds of singular nouns, such as employer, store, airport, website, school,
publisher,and neighborhood (e.g., I wanted to work at the shoe store, but they weren’t hiring). These nouns should be matched with
singular pronouns (e.g., I wanted to work at the shoe store, but it wasn’t hiring). If they, their, or them is underlined in a question,
make sure it refers to a plural noun or to multiple nouns. If it refers to a collective noun or some other singular noun, use  it (or he or
she if a person), not they.

(9) PRONOUN AGREEMENT: INDEFINITE PRONOUNS

   Indefinite pronouns such as everyone, everything, somebody, and anything are singular and take singular
pronouns.
Indefinite pronouns that refer to people, such as anyone, everybody, someone, somebody, or nobody, take the singular pronoun
constructionhe or she. Indefinite pronouns that refer to things, such as anything, something, or everything, take the singular
pronoun it. Also singular are the pronouns each and every.
The rules for pronoun use are similar to the rules of subject-verb agreement. Subjects that take singular verbs also take singular
pronouns. Subjects that take plural verbs also take plural pronouns.
Each of the students must carry his or her identification card on campus.
Everyone who works at the health clinic has had his or her flu shot.
Every student brought his or her calculator to the test.
Neither the principal nor the teachers brought their lunch to the meeting.
Neither city plans to renovate its train station.

TEST SAMPLE
Anyone who wants to try out for the tennis team on Saturday should be at the courts by 9:00 a.m., bring their own racquet,
and wear non-marking tennis shoes.

EXPLAINED
The plural pronoun their should not be used to refer to the indefinite pronoun anyone. Anyone is singular, so the pronoun
should be singular.

REVISED
Anyone who wants to try out for the tennis team on Saturday should be at the courts by 9:00 a.m., bring  his or her own
racquet, and wear non-marking tennis shoes.

(10) VAGUE PRONOUNS

   Which, this, that, these, those, it, and they must refer to something specific.


Be on alert for words such as it, this, that, and which. These words must refer to a specific noun in the sentence. They cannot refer
to an unnamed actor, a group of words, or an entire sentence.

TEST SAMPLE A
During the historic Apollo 11 mission, which is famous for Neil Armstrong’s walk on the moon, they collected twenty-two
kilograms of geologic material, including fifty lunar rocks.

EXPLAINED
The pronoun they does not refer to any specific noun in the sentence. One may assume that  they refers to the astronauts,
but a pronoun’s antecedent must be clear and explicit, not implied.

REVISED
During the historic Apollo 11 mission, which is famous for Neil Armstrong’s walk on the moon,  the astronauts collected
twenty-two kilograms of geologic material, including fifty lunar rocks.

TEST SAMPLE B
When Harold worked at Subway, he treated the customers rudely, forgot to bake the bread, and threw tomato slices at the
manager, which angered Subway’s owner.

EXPLAINED
What does which refer to? The pronoun reference is unclear. The pronoun which usually refers to the noun that precedes it,
but in this sentence which does not refer to the noun manager. The writer may have intended to use the pronoun which to
refer to all of Harold’s actions, but the pronoun which cannot modify a clause with so many elements. The
pronoun which must refer to a specific noun in the sentence.

REVISED
When Harold worked at Subway, he treated the customers rudely, forgot to bake the bread, and threw tomato slices at the
manager,actions that angered Subway’s owner.
OR: When Harold worked at Subway, he treated the customers rudely, forgot to bake the bread, and threw tomato slices at
the manager. These actions angered Subway’s owner.

TEST SAMPLE C
My mother wanted my brother Jerome to major in economics or finance and become an entrepreneur, but he never
demonstrated any aptitude for it.

EXPLAINED
What does it refer to? The sentence does not tell us. It must refer to a specific noun in the sentence.

REVISED
My mother wanted my brother Jerome to major in economics or finance and become an entrepreneur, but he never
demonstrated any aptitude for business.

TEST SAMPLE D
In response to a dare from his classmates, Sam threw an orange at the English teacher’s head and then pulled the fire
alarm. The principal reacted angrily, but Sam’s father thought this was hilarious.

EXPLAINED
What does this refer to? Does this refer to Sam’s behavior, to the principal’s anger, or to something else? The pronoun
reference is unclear.

REVISED
In response to a dare from his classmates, Sam threw an orange at the English teacher’s head and then pulled the fire alarm.
The principal reacted angrily, but Sam’s father thought Sam’s antics were hilarious.

(11) WHO IS FOR PEOPLE; WHICH IS FOR THINGS

   Who should be used to refer to people; which should be used to refer to things.

TEST SAMPLE
The army hospital is hiring doctors, nurses, and surgical assistants which have experience in emergency medicine and
rehabilitative care.

EXPLAINED
Doctors, nurses, and surgical assistants are people. The relative pronoun who, not which, should be used for nouns or
pronouns that refer to people.

REVISED
The army hospital is hiring doctors, nurses, and surgical assistants who have experience in emergency medicine and
rehabilitative care.
NOTE ABOUT WHO VS. THAT
Many grammar guides say that you can use either that or who for people (e.g., The army hospital hired a doctor who has experience
in emergency medicine OR The army hospital hired a doctor that has experience in emergency medicine). When referring to specific
people, however, use who, not that.
The army hospital hired Dr. Grimaldi, who has extensive experience in emergency medicine.

(12) WHICH OR THAT?

 That should be used to add information that is necessary. Which should be used to add information that is not necessary.
Students often have trouble understanding the difference between that and which. Both that and which are attached to nouns or
pronouns. Athat clause tells the reader something about the noun or pronoun that the reader needs to know to understand the
meaning of the sentence. Thethat clause answers the question “Which one?” or “Which ones?” in a sentence in which that
information is necessary. A which clause, like athat clause, adds information to the sentence, but the information added by
a which clause is not essential to the meaning of the sentence. You can remove a which clause and still understand the point of the
sentence.
The animal control officers are looking for the dogs that attacked our horse.
The relative clause that attacked our horse restricts the word dogs to include only those dogs that attacked the horse.
The clause is essential because it tells the reader which dogs the officers are looking for. Without that clause, the
sentence would have a different meaning.
Spencer collects typewriters that were once owned by famous authors.
The relative clause that were once owned by famous authors restricts the meaning of the word typewriters to include
only those typewriters that were once owned by famous authors. Without that clause, the sentence would suggest that
Spencer collects all typewriters.
Toccara bought a new car, which is blue.
The relative clause which is blue adds information, but that information is not essential. The point of the sentence is that
Toccara bought a new car. You can strike the which clause without changing the meaning of the sentence.
Omar prefers the pizza at Patty’s Pizza Emporium, which opened last year, to the pizza at Pizza Shack.
The relative clause which opened last year adds information, but that information is not essential. The clause can be
deleted without changing the meaning of the sentence. The point of the sentence is that Omar prefers the pizza at
Patty’s to the pizza at Pizza Shack.
Omar prefers the Pizza Shack that opened last year to any of the other Pizza Shacks.
In this sentence the relative clause that opened last year adds essential information. It identifies the specific Pizza Shack
that Omar prefers. The relative clause that opened last year answers the question “Which one?” Without the clause, the
sentence would be confusing: Omar prefers the Pizza Shack that opened last year to any of the other Pizza Shacks.
The difference between that and which is usually not tested directly in the identifying-sentence-errors section. Rather, it is tested
indirectly in the improving-sentences or improving-paragraphs section.

TEST SAMPLE
In the early twentieth century, unions urged Congress to enact laws that will end child labor in the United States.
(A) laws that will end
(B) laws; which will end
(C) laws that would end
(D) laws that will end
(E) laws, which would end
The answer is C. This question is designed, for the most part, to test a verb error. The verb should be  would, not will, because the
sentence looks forward from a point in the past. (This verb error is covered more fully in the section on would vs. will in the chapter
on verb errors.) But the answer choices also introduce a that-or-which problem. In this sentence, that is required because the
clause that [would] end child labor in the United States is essential to the meaning of the sentence. The that clause identifies which
laws the unions were urging Congress to pass. Without that clause, the sentence would have a different meaning.

(13) PRONOUN AGREEMENT: THAT VS. THOSE

     Use that when referring to singular nouns. Use those when referring to plural nouns.

TEST SAMPLE
Janice was warned by her pharmacist that the side effects of an herbal supplement may be just as dangerous as that of a
prescription drug or conventional over-the-counter medication.

EXPLAINED
What does that refer to? In this sentence the pronoun that refers to side effects. The sentence compares the side effects of
an herbal supplement to the side effects of other types of drugs.  Effects is plural. The singular pronoun that should be
changed to the plural pronoun those. If you had trouble figuring out what that refers to, try rephrasing the sentence to
include the noun that the pronoun refers to: Janice was warned by her pharmacist that the side effects of an herbal
supplement may be just as dangerous as the side effects of a prescription drug or conventional over-the-counter medication.

REVISED
Janice was warned by her pharmacist that the side effects of an herbal supplement may be just as dangerous as those of a
prescription drug or conventional over-the-counter medication.

(14) IMPROPER PRONOUN SHIFTS

     Pronouns should be consistent unless the context requires a shift. Don’t mix you and one,
one and they, or you and they.
A common error in the writing section is an improper shift in pronouns. In most cases, the pronouns in the sentence shift
from one to you, orthey to you, or one to they.

TEST SAMPLE
If you are a serious student of American history and enjoy visiting historic places, one should walk the Freedom Trail, a
paved walking trail that connects sixteen significant Revolutionary War sites in Boston.

EXPLAINED
This sentence has an unnecessary shift in pronouns from you to one.

REVISED
If you are a serious student of American history and enjoy visiting historic places, you should walk the Freedom Trail, a paved
walking trail that connects sixteen significant Revolutionary War sites in Boston.

(15) USING THE POSSESSIVE BEFORE A GERUND

 Use a possessive pronoun before a gerund.


Use the possessive pronoun (e.g., my, his, their), not the objective pronoun (e.g., me, him, they), before a gerund.

TEST SAMPLE A
strict enforcer of classroom etiquette, Ms. Marple told Bruno that she would no longer tolerate him interrupting her
lectures on great Elizabethan sonnets.
EXPLAINED
What is it that Ms. Marple will no longer tolerate? Is it Bruno or Bruno’s  interrupting? The context makes clear that Ms.
Marple is fed up with Bruno’s interrupting, not with Bruno himself. The pronoun him should be changed to the
pronoun his. Changing the pronoun tohis clarifies that it is Bruno’s interrupting that Ms. Marple will no longer tolerate.

REVISED
A strict enforcer of classroom etiquette, Ms. Marple told Bruno that she would no longer tolerate his interrupting her lectures
on great Elizabethan sonnets.

(16) APPOSITIVES: “WE THE PEOPLE”

   The presence of an appositive, either before or after the pronoun, does not change the case of the pronoun that
must be used.
Sometimes the question writers test your ability to spot pronoun errors by attaching the pronoun to something called an appositive.
You don’t need to know the term, but you do need to be able to spot an appositive. An appositive is a noun or noun phrase placed
next to another noun or noun phrase. The appositive and the noun describe the same thing, but the appositive describes the noun in
a different way. The appositive either precedes or follows the noun that it identifies. EXAMPLES (appositives in italics):
Bill Clinton, the forty-second President, attended the rally.
[The appositive the forty-second President merely renames Bill Clinton. They are the same person.]
Sylvia’s new dog, a brown terrier named Chuckles, is the one that bit Sally.
[The appositive a brown terrier named Chuckles identifies Sylvia’s new dog. They are the same thing.]
Sometimes the appositive includes a pronoun or attaches to a pronoun.
We  musicians appreciate a good sound system.
The new managers, Omar and  she, are unpopular with employees.
The Spanish teacher took the top three Spanish students—Claire, Martine, and me—to an Argentinean restaurant.
The presence of an appositive, either before or after the pronoun, does not change the case of the pronoun that must be used. If the
pronoun is a subject, use the subjective pronoun. If the pronoun is an object, use the objective pronoun.
The pronouns that cause the most trouble when attached to appositives are we and us. We should be used as a subject. Us should
be used as an object. Try striking out the appositive if you are unsure which case should be used.

TEST SAMPLE
The slow economy and lack of investment in new technologies have resulted in a shrinking job market for we software
engineers.

EXPLAINED
To test whether we is correct, drop the appositive software engineers: The slow economy and lack of investment in new
technologies have resulted in a shrinking job market for we. Striking the appositive reveals the error in pronoun
case. We should be us. Note also thatfor us engineers is a prepositional phrase. Objects of prepositional phrases must be in
the objective case.

REVISED
The slow economy and lack of investment in new technologies have resulted in a shrinking job market for  us software
engineers.

(17) WHO OR WHOM?

 Who is used as a subject; whom is used as an object. Use whom when it is the object of a preposition.


With the exception of the pattern in Example A below, the relative pronouns  who and whom are not usually tested on the SAT. We
cover these rules, however, to make sure you are prepared for any who-or-whom question that could appear on the test. The rules
for the pronouns whoand whom are the same as those that apply to other pronouns:
 Who should be used for the subject: the person who does the action of the verb.
 Whom should be used for the object: the person who receives the action of the verb.
 Pronouns that are the objects of prepositional phrases must be in the objective case, so prepositions should be followed
by whom,not who.
Though these rules seem simple, almost everyone has trouble applying them. One way to determine whether who or whom should
be used is to isolate the clause that has who or whom in it and substitute another pronoun pair, such
as he and him, for who and whom.

EXAMPLE A
Marcus is going to the beach with Britney and Keisha, each of [who] [whom] is a certified lifeguard.
Break the sentence into clauses:
Marcus is going to the beach with Britney and Keisha
each of [who] [whom] is a certified lifeguard
Isolate the who/whom clause:
each of [who] [whom] is a certified lifeguard
Substitute another pronoun pair:
Each of they is a certified lifeguard
Each of them is a certified lifeguard
Which is correct? The objective pronoun them is correct. Use whom.
Note also that whom is the object of the prepositional phrase of whom. The objects of prepositional phrases must be in the
objective case.
Each (of whom) is a certified lifeguard.

CORRECT
Marcus is going to the beach with Britney and Keisha, each of whom is a certified lifeguard.
TEST TIP: EACH OF WHOM, SOME OF WHOM, MANY OF WHOM
The pattern in Example A has been tested on the SAT, so the pattern is worth knowing. Phrases you should know include  all of
whom, some of whom, [number] of whom, most of whom, neither of whom, and each of whom.
Professor Dunkleman has seven teaching assistants, some of whom are National Merit scholars.
Harvey has six children, four of whom are career criminals.

EXAMPLE B
Dr. Haddad is the professor [who] [whom] is an expert in nanotechnology.
Break the sentence into clauses:
Dr. Haddad is the professor
[who] [whom] is an expert in nanotechnology
Isolate the who/whom clause:
[who] [whom] is an expert in nanotechnology
Substitute he and him:
he is an expert in nanotechnology
him is an expert in nanotechnology
Which is correct? The subjective pronoun he is correct. Use who.

CORRECT
Dr. Haddad is the professor who is an expert in nanotechnology.

EXAMPLE C
Twenty-five percent of voters have not decided [who] [whom] they will support in the general election.
Isolate the who/whom clause:
[who] [whom] they will support in the general election
Rearrange and substitute he and him:
They will support he in the general election.
They will support him in the general election.
Which pronoun is correct? Rephrasing shows that the objective case (him) should be used.

CORRECT
Twenty-five percent of voters have not decided whom they will support in the general election.

EXAMPLE D
The producer will choose the actor [who] [whom] has the best audition.
Isolate the who/whom clause:
[who] [whom] has the best audition
Rephrase and substitute:
he has the best audition
him has the best audition
Which pronoun is correct? He has the best audition is correct. The subjective case who should be used.

CORRECT
The producer will choose the actor who has the best audition.
NOTE ABOUT WHOEVER vs. WHOMEVER
You have already learned that whom should be used when it is the object of a preposition.
Tabitha must learn to tolerate people with whom she disagrees.
For whom is Jim working?
Billy is going to Hawaii to visit Uncle Bill, after whom he is named.
Dwayne has three daughters, two of whom are nuclear physicists.
But the rules are a bit different in sentences with whoever/whomever constructions. Consider the following example:
I will give my sweater to [whoever] [whomever] needs it.
You might be tempted to choose whomever because the pronoun appears to be the object of the prepositional phrase to [whoever]
[whomever]. But the object of the preposition in this sentence is the entire clause:  [whoever] [whomever] needs it. To determine
whether to use whoever or whomever, you should isolate that clause and try substituting a personal pronoun pair such
as he and him for whoeverand whomever.
Isolate the whoever/whomever clause: [whoever/whomever] needs it

Substitute he and him: he needs it

  him needs it

In this clause, he should be used. The correct pronoun is whoever.


I will give my sweater to whoever needs it.
Now consider the following sentence:
I will sell my coin collection to [whoever] [whomever] the dealer recommends.
Isolate the whoever/whomever clause:
[whoever] [whomever] the dealer recommends
Rearrange and substitute he and him:
The dealer recommends he.
The dealer recommends him.
The objective pronoun whomever is required.
I will sell my coin collection to whomever the dealer recommends.
It is highly unlikely that you will get a whoever-or-whomever question on the SAT, but we want you to be aware that the whoever-or-
whomever analysis is a bit different from the who-or-whom analysis.
REVIEW QUESTIONS: PRONOUNS

Check the following sentences for pronoun errors. If you find an error, correct it.
1. Jasmine drove her sister to the bridal salon so they could shop for her wedding dress.
2. For we movie stars, fame is a great burden.
3. If one wants to lose weight, they must limit their calorie intake and exercise regularly.
4. If anyone in the drama club wants to borrow materials from the art studio, they must seek written permission from the art
teacher.
5. One reason that eighty percent of the toys sold in the United States are manufactured in China is that the wages of
American workers are thirty times higher than that of Chinese workers.
6. The principal prefers to recruit teachers which have studied abroad and speak two or more languages.
7. Despite my best efforts, I cannot forgive Paul for heckling my mother at the poetry reading, and I now believe that the rift
between Paul and I will never heal.
8. Last week, the state assembly held their first session since returning from summer recess.
9. To build the Hong Kong International Airport, they built an island sixteen miles out at sea and constructed tunnels, bridges,
and roadways to connect the island to the mainland.
10. Judge Crater advised the jury that they should consider only the evidence presented at trial and render a fair and impartial
verdict.
11. Having left their uniforms at home, Bobby, Mariah, and him were banned from marching in the St. Patrick’s Day parade.
12. A foreign national who has been granted a visa to visit the United States must leave the United States before their visa
expires.
13. Having lost their federal funding, the research team had to decide whether to terminate the project or to seek private
funding.
14. When Murray goes to college next fall, him and his best friend will share an apartment close to campus.
15. Sven is my best friend, but I do not appreciate him calling me after midnight.
16. My grandmother has consulted with two orthopedic surgeons, neither of who recommends hip replacement surgery.
CHAPTER 3
VERBS

This chapter covers the verb errors you need to know for the SAT writing section. Though we identify the verb tenses by name, you
don’t need to know the names of the different verb tenses to identify and correct verb errors on the SAT. Focus on the examples and
test samples, not on the names of the tenses. The examples will make the rules clear.

(1) VERB TENSE: GENERAL RULE

     The verb tense must fit the time of the action.


The tense of a verb tells you when the action of the verb occurs. There are three basic tenses: present, past, and future.
SIMPLE TENSES

Present (action happening now or habitually) Jim Bob cries every day.

Past (action happened in the past) Jim Bob cried yesterday.

Future (action will happen in the future) Jim Bob will cry tomorrow.

In addition to the simple tenses, there are progressive tenses, perfect tenses, and perfect-progressive tenses, each of which has a
present, past, and future form. These tenses allow the writer to be more specific about the time of the action or to show the
relationship in time between different actions or conditions.
PROGRESSIVE TENSES

Present-progressive Jim Bob is crying again.

Past-progressive Jim Bob was crying yesterday.

Future-progressive Jim Bob will be crying by the time the movie ends.

PERFECT TENSES

Present-perfect Jim Bob has cried every day for two weeks.

Past-perfect After Jim Bob had cried all day, he blew his nose.

Future-perfect By the time the movie ends, Jim Bob will have cried for two hours.

PERFECT PROGRESSIVE TENSES

Present-perfect progressive Jim Bob has been crying since the movie started.

Past-perfect progressive Jim Bob had been crying when I arrived.

Future-perfect progressive Jim Bob will have been crying for three days by the time his mother arrives.

You don’t need to be an expert in verb tenses to identify verb errors on the SAT. Sentences that have verb errors usually provide
clues about the time of the action. Phrases such as in 1960, last month, in the early days of television, and during the Renaissance tell
you that the action took place in the past. Phrases such as next September, in future generations, and later this year tell you that the
action will occur in the future. If a verb is underlined in a test question, make sure that the tense of the verb matches the time of the
action. Unlike almost all other SAT errors, verb-tense errors often “sound wrong.”
TEST SAMPLE A
In 1767, the Parliament of Great Britain passed a series of acts known as the Townshend Acts, which will tax tea, paper, and
other products imported into the colonies.

EXPLAINED
The sentence describes actions that took place in 1767. The verbs should be in the past tense. The future tense will tax is
incorrect.

REVISED
In 1767, the Parliament of Great Britain passed a series of acts known as the Townshend Acts, which taxed tea, paper, and
other products imported into the colonies.

TEST SAMPLE B
In the nineteenth century, wildlife biologists in Australia are surprised to learn that the odd-looking platypus has a
venomous spur.

EXPLAINED
The sentence describes something that occurred in the nineteenth century. The present tense (biologists . . . are
surprised) does not make sense in this context. The past tense were should be used.

REVISED
In the nineteenth century, wildlife biologists in Australia were surprised to learn that the odd-looking platypus has a
venomous spur.
Did you notice that the present-tense has is used in the last part of the sentence? The clause the odd-looking
platypus has a venomous spur states a scientific fact. Statements of scientific fact are stated in the present: My brother
did not know that water boilsat 212 degrees Fahrenheit.

TEST SAMPLE C
Yesterday, Senator Levin told reporters that he opposed the appropriations bill when it comes up for a vote next week.

EXPLAINED
In this sentence, the verbs told and opposed are both in the past tense, but they shouldn’t be. The first verb, told, is properly
in the past tense: Senator Levin told the reporters yesterday. But opposed is not the correct verb to use when referring to
what the senator will do next week. The senator will oppose the bill next week. A shift in verb tense is necessary.

REVISED
Yesterday, Senator Levin told reporters that he will oppose the appropriations bill when it comes up for a vote next week.

(2) IMPROPER SHIFTS IN VERB TENSE

     Verb tenses should be consistent. Tenses should shift only if the context requires a shift.
Shifts in verb tense are often necessary to show a shift in time. Consider the following example:
My brother is currently performing in Spamalot, a Broadway musical based on the film Monty Python and the Holy
Grail, which was originally released in 1975.
The shift in verb tense from the present progressive to the past is necessary because the two actions occurred at different times: my
brother is performing in the play now, but the film was released in 1975.
Remember, however, that verb tenses should shift only if the context requires a shift. If a sentence describes two or more actions
that occurred in the same general time frame and nothing in the sentence requires a shift in tense, the verb tenses should be
consistent.

TEST SAMPLE
As late as the mid-twentieth century, public schools had strict dress codes for students, parents signed every homework
assignment, and teachers are using physical force to discipline students.
EXPLAINED
This sentence has an illogical shift in verb tense. The sentence describes three actions that occurred concurrently in the past.
All three verbs should be in the past tense.

REVISED
As late as the mid-twentieth century, public schools had strict dress codes,
parents signed every homework assignment, and teachers used physical force to discipline students.

(3) IRREGULAR VERBS

     Has, have, and had must be followed by the past participle (e.g., gone), not the past tense (e.g., went), of
irregular verbs.
Most verbs can be changed from the present tense to the past tense or the past participle by adding -ed to the end of the verb.

PRESENT PAST PAST PARTICIPLE (used with has, have, or had)

look looked looked

borrow borrowed borrowed

call called called

smile smiled smiled

Irregular verbs do not follow this rule. The following is a chart showing the present tense, past tense, and past participle of a few
common irregular verbs:

PRESENT PAST PAST PARTICIPLE (used with has, have, or had)

eat ate eaten

go went gone

arise arose arisen

begin began begun

swim swam swum

drink drank drunk

see saw seen

Some test questions mix these tenses to create incorrect verb forms such as has went, have arose, has drank, and have swam.

TEST SAMPLE
The steep decline in home prices, coupled with the tightening of the credit market, has shook the confidence of real estate
brokers, who only two years ago were selling homes at record high prices.

EXPLAINED
The simple past tense of shake is shook. The past participle of shake is shaken. Has shook is incorrect. The correct verb form
is has shaken.

REVISED
The steep decline in home prices, coupled with the tightening of the credit market, has shaken the confidence of real estate
brokers, who only two years ago were selling homes at record high prices.
TEST TIP: LOOK FOR THE HELPING VERB
You might think that this type of error is easy to spot, but question writers often insert words between the helping verb  has,
have, or hadand the past participle.
Disputes have frequently arose over the school’s cell phone policy. [correct verb: have arisen]
Emilio has occasionally drove through the mountain pass. [correct verb: has driven]
Shifty has never swam the length of the pool. [correct verb: has swum]
The key is to look for the helping verb that signals the need for the past participle.
For additional examples of irregular verbs, see Appendix A.

(4) VERBS THAT MUST BE FOLLOWED BY AN INFINITIVE

   Some verbs must be followed by an infinitive (e.g., to go), not a gerund (e.g., going).


Some verbs, such as agree, plan, promise, or choose, must be followed by an infinitive (e.g., to go, to run, to beg, to leave). A
common error on the SAT is the placement of a gerund (e.g., going) after a verb that requires an infinitive (to go).

WRONG Carmelo plans running for political office as soon as he raises enough money.

RIGHT Carmelo plans to run for political office as soon as he raises enough money.

WRONG Harold’s mother promised taking Harold to Disneyland as soon as he completes his anger management therapy.

RIGHT Harold’s mother promised to take Harold to Disneyland as soon as he completes his anger management therapy.

Some of the other verbs that require an infinitive are decide, expect, forget, hope, prepare, refuse, and want.

TEST SAMPLE
Although he was offered a full scholarship to the University of Alaska, Bertram chose enrolling at the University of Hawaii.

EXPLAINED
The verb chose should not be followed by the gerund enrolling. Instead, the verb chose should be followed by the infinitive to
enroll.

REVISED
Although he was offered a full scholarship to the University of Alaska, Bertram chose to enroll at the University of Hawaii.

(5) PERFECT TENSES: HAD WORKED OR HAS WORKED?

   The past perfect (e.g., had worked) should be used for actions that started in the past and ended in the past. The
present perfect (e.g., has worked or have worked) should be used for actions that started in the past and continue in the
present.
There is a difference between the present-perfect tense and the past-perfect tense. The present-perfect tense (e.g., has
worked or have worked) describes an action that occurred at an unspecified time in the past, or an action that started in the past and
continues in the present.
Action occurred at an unspecified time in the past:
Angelina has worked as a park ranger.
Omar has climbed Mount Rainier.
Action started in the past and continues in the present:
Paris and Nicky have worked at the bank since they graduated from college.
Mr. Bean has taught at the local high school for most of his career.
Marcus has studied psychology for three years.
The past-perfect tense (e.g., had worked), in contrast to the present perfect, is used to describe an action that started in the past,
continued for some time in the past, and then ended in the past, usually when some other event occurred.
Paris had worked at the bank before the robbery.
Mr. Bean had taught Latin before the school eliminated the classics department.
Marcus had studied history before he studied psychology.
One error that sometimes appears on the SAT is the use of the past perfect (e.g., had worked) in a sentence that describes an action
that is ongoing or a condition that still exists. In such sentences, the present perfect (has worked) should be used.

TEST SAMPLE A
For more than a hundred years, the Statue of Liberty had stood on Liberty Island in New York Harbor, welcoming
immigrants and visitors to the United States.

EXPLAINED
Had stood suggests that the Statue of Liberty used to stand in the harbor but no longer stands there. The verb should be
changed to has stood, which makes clear that the Statue of Liberty still stands in the harbor today.

REVISED
For more than a hundred years, the Statue of Liberty has stood on Liberty Island in New York Harbor, welcoming immigrants
and visitors to the United States.
Another error that sometimes appears on the SAT is the use of the present tense or the past tense in a sentence that requires one of
the present-perfect tenses. The clue in many of these sentences is the use of the word since to express time (not as a substitute
for because).Examples are since 1970, since the earthquake, since noon, and since they returned from Chile. These phrases set the
time frame in the sentence from a specific point in the past through the present and require one of the present-perfect tenses.

WRONG Lamar, the new poultry expert at Safeway, worked as a butcher since 2003.

RIGHT Lamar, the new poultry expert at Safeway, has worked as a butcher since 2003.

  OR: Lamar, the new poultry expert at Safeway, has been working as a butcher since 2003.

WRONG Corky knows Alex since they were classmates in kindergarten.

RIGHT Corky has known Alex since they were classmates in kindergarten.

WRONG Lemuel took classes at the university since he returned from Iraq.

RIGHT Lemuel has been taking classes at the university since he returned from Iraq.

TEST SAMPLE B
Since 1995, former Nirvana drummer Dave Grohl was the lead singer and songwriter for the Foo Fighters.

EXPLAINED
The simple past tense was should be the present perfect has been. Dave Grohl formed the Foo Fighters in 1995 and
continues to be its lead singer and songwriter. The condition is ongoing; therefore, the present-perfect tense is required.

REVISED
Since 1995, former Nirvana drummer Dave Grohl has been the lead singer and songwriter for the Foo Fighters.
NOTE ABOUT SINCE
Since does not always require the present-perfect tense. For example, if you were writing Lemuel’s obituary, you could
say Lemuel hadbeen a butcher since 1993. The past perfect is appropriate because Lemuel stopped being a butcher when he died.
The SAT questions that test the perfect tenses, however, almost always use since to describe a condition that started in the past and
continues in the present. The question will either refer to something you know still exists (e.g., the Grand Canyon, the Mississippi
River, the Golden Gate Bridge) or include other information that makes the time frame clear (e.g., Viswanathan Anand, an Indian
grandmaster who lives in Spain, has beenthe world champion of chess since 2007).
(6) “IF I HAD, I WOULD HAVE”

 In sentences that express what would have happened if things had been different, follow the pattern if I had, I would
have.
Some sentences on the test express what would have happened if things had been different. These sentences usually have
an if clause and a consequences clause: If I had read the book, I would have written a better book report. In these types of sentences,
do not use would have in the if clause; use had. Would have or could have may be used in the consequences clause. Follow the
pattern If I had, I would have, or I would have if I had.

TEST SAMPLE A
Spending another day in summer school, instead of at the beach, Harold is reminded that he would have graduated with his
classmates if he would have completed his senior project.

EXPLAINED
Do not use would have in an if clause. “If he would have completed” should be “if he had completed.”

REVISED
Spending another day in summer school, instead of at the beach, Harold is reminded that he would have graduated with his
classmates if he had completed his senior project.

TEST SAMPLE B
We enjoy Carla’s company, but if Carla would have gone with us to Florida, she would have watched soap operas in the
hotel room all day.

EXPLAINED
Do not use would have in an if clause. Would have in the if clause should be had.

REVISED
We enjoy Carla’s company, but if Carla had gone with us to Florida, she would have watched soap operas in the hotel room
all day.

(7) “IF I WERE A MOVIE STAR” AND OTHER HYPOTHETICAL STATEMENTS

 In sentences that express a hypothetical or speculate about a condition that is contrary to fact, follow the pattern  if I
were, I would.
In an if clause that expresses a hypothetical or speculates about a condition that is contrary to fact, use  were in the if clause
and would in the consequences clause.
If I were a movie star, I would buy a mansion in the Hollywood Hills.
If Omar were a professional hockey player, he would be a goalie for the Flyers.
If I were a contestant on American Idol, I would sing “Crazy” by Gnarls Barkley.
I would move to another town if Arnie were the police chief.

(8) WILL OR WOULD?

   When something may happen in the future, use would. When something will definitely happen in the future,
use will.
Would is a flexible word that may be used in many different ways. For example,  would is used in sentences that look forward from a
point in the past: In the early twentieth century, unions urged Congress to enact laws that would end child labor in the United
States. The future in that sentence is a future of the past. Would is also properly used in sentences that express a hypothetical or a
wish.
If I had a million dollars, I would buy a jet.
If I were the principal, I would allow students to use cell phones in class.
Will, in contrast, is used to express certainty. If a sentence describes an event that is certain to occur immediately or in the future
and is not subject to any condition, use will, not would.

TEST SAMPLE
Although the conference on Asian trade ends today, the governor would stay in China until next Wednesday to inspect
several factories that export food products to the United States.

EXPLAINED
This sentence describes the governor’s immediate plans, which are certain and not subject to condition. The use of  would is
incorrect. The correct verb is will.

REVISED
Although the conference on Asian trade ends today, the governor will stay in China until next Wednesday to inspect several
factories that export food products to the United States.

(9) PARTICIPLES: STANDING OR HAVING STOOD?

 Use the present participle (e.g., standing) when referring to an action that occurred at the same time as the main action
in the sentence. Use the present-perfect participle (e.g., having stood) when referring to an action that occurred before the main
action in the sentence.
Participles are verb forms that function as adjectives. Like other verb forms, participles have tenses: the present participle
(e.g., walking), the past participle (walked), and the present-perfect participle (having walked). Many sentences start with participial
phrases, which are phrases that contain participles. The tense of the participle often shows the sequence of the events described in
the sentence.
Standing on the balcony, Harlan dropped a water balloon.
[The present participle walking is used to show that the two actions in the sentence occurred at the same time. Harlan
was standing on the balcony when he dropped a water balloon.]
Having dropped the water balloon, Harlan ran into the apartment.
[The present-perfect participle having dropped shows that the action in the participial phrase occurred before the action
in the main clause: Harlan dropped the water balloon first and then ran into the apartment.]
Angered by Harlan’s actions, Harlan’s father ordered Harlan to stay indoors.
[In this sentence, the past participle angered merely explains the action in the main clause: Harlan’s father was angry
about the incident, so he ordered Harlan to stay indoors.]
Some questions on the SAT test your ability to spot tense errors in participial phrases. Most of these questions require you to choose
between the present participle (e.g., standing) and the present-perfect participle (e.g., having stood). You should select the present
participle (e.g.,standing) when the action in the participial phrase takes place at the same time as the action in the main clause. You
should choose the present-perfect participle (e.g., having stood) when the action in the participial phrase takes place before the
action in the main clause. Here are a couple of other examples:
Walking on the beach, Billy Bob spotted a crab.
[The two actions occurred at the same time: Billy Bob was walking on the beach when he spotted a crab.]
Having taken marine biology in college, Billy Bob was able to identify the crustacean as a ghost crab.
[One action occurred before the other: Billy Bob took marine biology first and then was able to identify the crab.]

TEST SAMPLE
The football team beating every opponent during the regular season, they were shocked to lose the championship game by
27 points.
A. The football team beating every opponent during the regular season, they were
B. Beating every opponent during the regular season, the football team was
C. Having beaten every opponent during the regular season, the football team was
D. To beat every opponent during the regular season, the football team were
E. The football team beat every opponent during the regular season, and they were
The correct answer is C. Choices B and C are similar. The difference between the two choices is the tense of the verb in the participial
phrase. Two events are described in the sentence: (1) the team beat every opponent during the regular season, and (2) the team
was shocked to lose the championship game. The regular-season wins occurred before the shock of losing the championship game,
so the present-perfect participle, having beaten, not the present participle, beating, is the correct participle to use. The original
sentence also has a pronoun error.Team is a collective noun, which is singular. Team requires the singular pronoun it, not the plural
pronoun they. Choice C drops the faulty pronoun and also correctly pairs the singular noun team with the singular verb was.

REVIEW QUESTIONS: VERBS

Review the following sentences for verb errors. If there is an error, correct it.
1. At last Tuesday’s city council meeting, the mayor explains his opposition to naming City Hall after the former governor, who
is currently serving a ten-year sentence for bribery.
2. NASA’s new director would give a tour of the new control room to reporters tomorrow at noon.
3. Disputes between administrators and teachers have frequently arose over the usefulness of standardized tests as a tool for
measuring student performance.
4. A national landmark, the Golden Gate Bridge had connected San Francisco to Sausalito since 1937.
5. I never would have applied to this university if someone would have told me that dating, snacking, and napping are
prohibited.
6. One of nine children, Grover Cleveland was born in New Jersey in 1837, was raised in upstate New York, and starts his
political career as mayor of Buffalo.
7. Last week the manager told his employees that he has been quitting his job by the end of next month.
8. By the time Denzel entered college in 1982, he had written a Broadway play, had driven across the country twice, and had
swam from Sausalito to San Francisco.
9. While Harold was in Hawaii at a Star Trek convention, his brother Mark is in Chicago at an auto show.
10. Rapper MC Vanilla, whose song “Nice, Nice Baby” topped the Billboard charts in 1984, plans appearing at the MTV Music
Awards to dispel rumors that he is living with an indigenous tribe in South America.

CHAPTER 4
PARALLEL STRUCTURE

(1) PAIRS AND SERIES: HIKING, BIKING, AND SWIMMING

     Similar elements in a sentence should follow the same grammatical pattern (to go, to run, and to
hide; not to go, to run, and hiding).
Parallelism requires that similar elements in a sentence be phrased in the same way. Sentences that mix different constructions are
grammatically incorrect. When you see two or more words or phrases connected by and or or, make sure they are phrased in the
same way.
Billy Bob’s favorite pastimes are skiing and hiking.
The mechanic needs to tune the engine, change the oil, and rotate the tires.
This morning, I ate breakfast, ran a mile, and watched the news.

TEST SAMPLE A
The park ranger advises hikers to register at the ranger station, to bring a compass and a current map of the trail, and bright
colors should be worn during hunting season.

EXPLAINED
The park ranger advises hikers:
(1) to register at the ranger station,
(2) to bring a compass and a current map of the trail, and
(3) bright colors should be worn during hunting season.
The three elements in the series are not parallel.

REVISED
The park ranger advises hikers to register at the ranger station, to bring a compass and a current map of the trail, and to
wear bright colors during hunting season.

TEST SAMPLE B
The editors will evaluate each written submission based on content, style, and how original it is.

EXPLAINED
The elements in the series (content, style, and how original it is) are not in the same form.

REVISED
The editors will evaluate each written submission based on content, style, and originality.

TEST TIP: PROBLEMS WITH ARTICLES, INFINITIVES, AND PREPOSITIONS

Question writers often create errors in parallelism by omitting or misusing articles, infinitives, and prepositions.
(1) ARTICLES. The article that precedes the series (a, an, or the) must apply to all the elements in the series. If it doesn’t, use the
appropriate article for each element in the series.
WRONG an iguana, alligator, and lizard

  [The article an cannot be used with lizard.]

RIGHT

an iguana, an alligator, anda lizard

(2) INFINITIVES. If the words in the series are infinitives (e.g., to run, to hide), the word to should be placed either before the series
or before each of the verbs in the series.
WRONG to walk, to run, and skip

RIGHT to walk, to run, and to skip

  OR: to walk, run, and skip

(3) PREPOSITIONS. If the elements in the series require different prepositions, make sure each preposition is present in the series.
WRONG Mark disapproves and protests against using animals in medical research.

  [Disapproves needs its own preposition; it does not take against.]

RIGHT Mark disapproves of and protests against using animals in medical research.

(2) CONJUNCTION PAIRS: NEITHER BIKING NOR HIKING

     Sentences with conjunction pairs must be grammatically balanced (neither coming nor going, not neither
coming nor to go).
Certain conjunctions appear in pairs:
both . . . and
either . . . or
neither . . . nor
not only . . . but also
whether . . . or
The phrasing that follows the first half of the conjunction pair must match the phrasing that follows the second half.

TEST SAMPLE A
Having lost its federal funding, the research team had to decide whether to terminate the project or if private funding
should be sought.

EXPLAINED
This sentence has the conjunction pair whether . . . or. The phrases that follow each conjunction should be balanced
grammatically, but they are not.
whether to terminate
or if private funding should be sought
The halves are not in the same form.

REVISED
Having lost its federal funding, the research team had to decide whether to terminate the project or to seek private funding.

TEST SAMPLE B
Mike Myers is both a brilliant physical comedian and has been successful at screen writing.

EXPLAINED
This sentence includes the conjunction pair both . . . and. The phrases that follow each conjunction should be balanced
grammatically, but they are not.
both a brilliant comedian
and has been successful at screen writing

REVISED:
Mike Myers is both a brilliant physical comedian and a successful screenwriter.

TEST SAMPLE C
Jessica has neither the athletic ability to succeed in sports nor does she have the motivation to succeed in school.

EXPLAINED
This sentence has the conjunction pair neither . . . nor. The phrases that follow each conjunction should be parallel, but they
are not.
neither the athletic ability to succeed
nor does she have the motivation to succeed

REVISED
Jessica has neither the athletic ability to succeed in sports nor the motivation to succeed in school.

TEST TIP: EVERY WORD COUNTS!

The misplacement of a single word can create an error in parallelism. Beware of this error on the SAT. If you see a conjunction pair
such as both . . . and, or either . . . or, check for parallelism.
WRONG Soccer is popular both in Chile and Argentina.

  Is this sentence parallel? No. The first half of the conjunction pair has a preposition but the second does not.

      both   in Chile
      and    Argentina
RIGHT Soccer is popular both in Chile and in Argentina.
WRONG Yoga class will be held in either the studio or community center.

  Is this sentence parallel? No. The first half of the conjunction pair has an article (the), but the second half does not.

      either   the studio
      or     community center
  The appropriate article (a, an, or the) must be added.

RIGHT Yoga class will be held in either the studio or the community center.

TEST SAMPLE D
Harmon will attend the convention as either a delegate or as an organizer.

EXPLAINED
The conjunction pair in this sentence is either . . . or. What follows either should match what follows or. Do they match?
either a delegate
or as an organizer
The elements do not match because of the placement of the word as. The sentence needs to be rephrased.

REVISED
Harmon will attend the convention either as a delegate or as an organizer.
OR: Harmon will attend the convention as either a delegate or an organizer.

(3) COMPARING AND CONTRASTING: RUNNING IS HARDER THAN WALKING

   Sentences that compare and contrast must be parallel.


Some sentences compare or contrast two things or ideas. These sentences usually include words of comparison such as as or than:

More A than B = Manny relies more on his memory than on his class notes.

  Manny likes skiing more than hiking.

Less A than B = Marco is interested less in his studies than in his social life.

A rather than B = Selena would rather stay home than go to school.

  My teacher told me that I should read a chapter each week rather than read the whole book in one night.

Sentences that compare and contrast two elements or ideas must be parallel. Think about parallel structure as balance. Whatever
phrasing is used on one side of the comparison must be used on the other side.

TEST SAMPLE
Many movie stars say that performing on Broadway is far more challenging and rewarding than their experience as actors in
films.

EXPLAINED
This sentence contains a comparison, but the elements in the comparison are not parallel. Performing on Broadway is not
phrased in the same way as their experience as actors in films. Remember that comparisons require balance.

REVISED
Many movie stars say that performing on Broadway is far more challenging and rewarding than acting in films.

REVIEW QUESTIONS: PARALLEL STRUCTURE


Review the following sentences for any errors in parallel structure. If there is an error in parallelism, correct it.
1. Issued on January 1, 1863, the Emancipation Proclamation committed the Union to ending slavery, freed all the slaves in
the Confederacy, and African-Americans were authorized to enlist in the Union army.
2. Neither hiking in the Appalachian Mountains nor the dive in the Gulf of Mexico was as much fun as I thought it would be.
3. Candidates for political office must decide whether to use public money to fund their campaigns or if they should rely on
private contributions to pay campaign expenses.
4. Sean Combs, known in the music world as Diddy, is both a successful record producer and, as an entrepreneur, he owns a
clothing line, film-production company, and two restaurants.
5. Professor Presley is interested more in writing his book about bees than to assist students with their lab work.
6. To qualify for an Oscar nomination, an animated film must run at least seventy minutes, have more animated action than
live action, and a frame-by-frame technique must be used to create movement.
7. While studying in Egypt in 2002, Jennifer both visited popular tourist destinations and obscure historical sites.

CHAPTER 5
COMPARISONS

(1) APPLES TO APPLES: COMPARISONS MUST BE LOGICAL

     Comparisons must be logical (e.g., comparing Kanye West’s music to Eminem’s music, not Kanye West’s
music to Eminem).
Comparison errors are frequently tested, but they can be difficult to spot. Look for words in the sentence that signal a comparison
between two things (e.g., as enjoyable as, more careful than, unlike her sister) and ask yourself if the two things can be logically
compared.

TEST SAMPLE A
Unlike her brother Simon, who follows a low-fat, vegetarian diet and exercises regularly, Helga’s diet consists mostly of
donuts and cheeseburgers, and she refuses to work out.

EXPLAINED
This pattern frequently appears on the test. Always look for phrases such as like her sister or unlike sea snakes. These phrases
signal comparisons. In this sentence, her brother Simon is being compared to Helga’s diet. That comparison is illogical. The
sentence should be rephrased to compare Simon to Helga, or Simon’s diet to Helga’s diet, but not Simon to Helga’s diet.

REVISED
Unlike her brother Simon, who follows a low-fat, vegetarian diet and exercises regularly, Helga eats mostly donuts and
cheeseburgers, and she refuses to work out.

TEST SAMPLE B
One reason that eighty percent of the toys sold in the United States are manufactured in China is that the wages of
American workers are thirty times higher than China.

EXPLAINED
The words higher than signal a comparison. If you read the sentence closely, you will see that the wages of American workers
are being compared to the country of China. That comparison is illogical. The sentence should be rephrased to compare the
wages of American workers to the wages of Chinese workers.

REVISED
One reason that eighty percent of the toys sold in the United States are manufactured in China is that the wages of American
workers are thirty times higher than those of Chinese workers.

TEST SAMPLE C
Like the elf owl, the nests of the Gila woodpecker are built in saguaro cacti in the Sonoran desert.

EXPLAINED
The phrase like the elf owl signals a comparison. The sentence, as written, compares the elf owl to the nests of a woodpecker.
That comparison is illogical. The comparison should be bird to bird, or nest to nest, but not bird to nest.

REVISED
Like the nests of the elf owl, the nests of the Gila woodpecker are built in saguaro cacti in the Sonoran desert.
OR: Like the elf owl, the Gila woodpecker builds its nests in saguaro cacti in the Sonoran desert.

TEST SAMPLE D
My mother’s chocolate cream pie is easier to prepare and tastes better than Martha Stewart.

EXPLAINED
This sentence compares my mother’s chocolate cream pie to Martha Stewart. The comparison is illogical. The comparison
should be pie to pie, or person to person, but not pie to person.

REVISED
My mother’s chocolate cream pie is easier to prepare and tastes better than Martha Stewart’s chocolate cream pie.
OR: My mother’s chocolate cream pie is easier to prepare and tastes better than Martha Stewart’s.

TEST SAMPLE E
Leonard’s mixed feelings about leaving home are like many other college freshmen: he craves more freedom but he fears
more responsibility.

EXPLAINED
This sentence is tricky. The words are like signal a comparison. As written, the sentence compares Leonard’s mixed
feelings with many other freshmen. The comparison is illogical. Leonard’s feelings should be compared to the feelings of
other freshmen.

REVISED
Leonard’s mixed feelings about leaving home are like those of many other college freshmen: he craves more freedom but he
fears more responsibility.

(2) DEGREES OF COMPARISON: BETTER OR BEST?

   Words such as better, greater, and smarter should be used to compare two things. Words such as best,
greatest, and smartestshould be used to compare three or more things.
Adjectives have degrees of comparison:
good, better, best
lazy, lazier, laziest
competent, more competent, most competent
If a sentence compares two things, use the comparative form (e.g., better, lazier, more competent). If the sentence compares three
or more things, or identifies something as the top in a class or category, use the superlative form (e.g.,  best, laziest, most
competent).

TEST SAMPLE
Although both leopards and jaguars are spotted felines known for their strength and swiftness, the jaguar is the
strongest and fastest of the two big cats.

EXPLAINED
The sentence compares the strength and speed of two big cats. Use the comparative, not the superlative, in sentences
that compare two things.
REVISED
Although both leopards and jaguars are spotted felines known for their strength and swiftness, the jaguar is
the stronger and faster of the two big cats.

(3) MISUSE OF MORE AND MOST

   Do not combine two different forms of the comparative or superlative. Remember that Muhammad Ali is known as
“The Greatest,” not “The Most Greatest.”
Most adjectives have a comparative and a superlative form. For many adjectives, the comparative is formed by adding  -er to the end
of the adjective, and the superlative is formed by adding -est:

wide wider widest

cruel crueler cruelest

weird weirder weirdest

crazy crazier craziest

For other adjectives, more is placed before the adjective to form the comparative, and most is placed before the adjective to form
the superlative:

cautious more cautious most cautious

brilliant more brilliant most brilliant

One error tested on the SAT is the combination of both forms (e.g., more quicker, most prettiest, more larger). These forms should
never be combined. Use quicker, NOT more quicker. Use largest, NOT most largest.

TEST SAMPLE
The readers of Sports Illustrated voted Carl Lewis the most greatest runner of the last century.

EXPLAINED
The superlative form of great is greatest. More or most should not be added to words that are already in the
comparative or superlative form. Carl Lewis is the greatest runner, not the most greatest.

REVISED
The readers of Sports Illustrated voted Carl Lewis the greatest runner of the last century.

REVIEW QUESTIONS: COMPARISONS

Review the following sentences for comparison errors. If the sentence has an error, correct it.
1. Unlike Martha Stewart, who uses pork to make chili, my mother’s chili is made with beef.
2. Although Barack Obama was only forty-seven when he was inaugurated and Bill Clinton was forty-six, John F. Kennedy, at
forty-three, was the most youngest President of the United States.
3. Eating dinner in Spain is not like the United States: Americans typically eat dinner around 6:00 p.m., but Spaniards eat
dinner after 9:00 p.m., often close to midnight.
4. Professor Ogletree selected Monique as his research assistant because she was the most experienced of the two
candidates.
5. Burger Boy’s new advertising campaign boasts that the hamburgers at Burger Boy are juicier, fresher, and more nutritious
than Burger World.
6. The owners spent over $150,000 to remodel their restaurant so that the tables in the back of the room would become as
desirable as, if not more desirable than, the front of the room.
7. Jim Bob was relieved that the school district sent all of his truancy notices to his mother, whose punishments were always
shorter and less severe than his father.
8. Of the three most widely used grammar guides, Jane Straus’s Blue Book of Grammar and Punctuation is more clearly
written and easier to use.

CHAPTER 6
MODIFIERS

(1) MISPLACED MODIFIERS

     Modifiers should be placed as close as possible to the word or words they modify and where they do not
create confusion or ambiguity.
A modifier is a word or a phrase that modifies something else in the sentence. In everyday speech, we are careless with modifiers.
We say, for example, “I only ate a cupcake for lunch today.” In that sentence, the word only modifies the verb ate, suggesting that
the speaker considered other things he or she could have done to the cupcake but decided only to eat it. The modifier  only is meant
to modify cupcake and should be moved as close as possible to that word: “I ate only a cupcake for lunch today.”
Although misplaced-modifier errors can be tested in many ways, there are three particular error patterns to look for on the test:
(1) relative clauses attached to the wrong noun;
(2) descriptive phrases attached to the wrong noun; and
(3) verb phrases (e.g., Having practiced all year, Walking on the beach, Circling the boat) that are not placed next to the name of
the person or thing doing the action of the verb.

TEST SAMPLE A
“The Battle Hymn of the Republic” became one of the most popular songs of the Civil War era, which was written by
Julia Ward Howe in 1862.

EXPLAINED
The Civil War era was not written by Julia Ward Howe. The relative clause which was written by Julia Ward Howe in
1862 should be placed next to the noun it modifies, which is “The Battle Hymn of the Republic.”

REVISED
Written by Julia Ward Howe in 1862, “The Battle Hymn of the Republic” became one of the most popular songs of the
Civil War era.
OR: “The Battle Hymn of the Republic,” written by Julia Ward Howe in 1862, became one of the most popular songs of
the Civil War era.

TEST SAMPLE B
On a safari through Kruger National Park in South Africa, Daisy observed the elephants and lions wearing binoculars.

EXPLAINED
Were the elephants and lions wearing binoculars? The phrase wearing binoculars is misplaced.

REVISED
On a safari through Kruger National Park in South Africa, Daisy wore binoculars to observe the elephants and lions.

TEST SAMPLE C
Racing at speeds between 130 and 160 miles per hour, large crowds gathered to watch the hydroplanes.

EXPLAINED
The phrase racing at speeds between 130 and 160 miles per hour is incorrectly linked to large crowds. The hydroplanes,
not the large crowds, were racing at high speeds. The phrase should be connected to the word it modifies (hydroplanes).
REVISED
Racing at speeds between 130 and 160 miles per hour, the hydroplanes attracted large crowds.

(2) DANGLING MODIFIERS

     A modifying phrase must have a noun to modify. The noun must be specifically and clearly named. It cannot
be implied.
Dangling-modifier errors appear frequently on the test, but they can be difficult to spot. Look for sentences that have an action
phrase but do not specifically and clearly name the doer of the action, or sentences that have a descriptive phrase but do not
specifically and clearly name the person or thing being described.

WRONG
Having passed the driving test, the state issued a license.
[This sentence does not identify who passed the driving test. The modifier dangles because the person doing the action
is not named in the sentence.]

RIGHT
Having passed the driving test, Miguel was issued a license.

WRONG
Having received critical acclaim for his role in Thelma and Louise, Brad Pitt’s acting career soared.
[Who received critical acclaim for his role in Thelma and Louise? You might think it is Brad Pitt, but the sentence does
notspecifically and clearly name Brad Pitt as the person who received critical acclaim. Instead, it names Brad
Pitt’s career as the subject. This is an error, and it is one that frequently appears on the test.]

RIGHT
After Brad Pitt received critical acclaim for his role in Thelma and Louise, his acting career soared.
If a sentence begins with an action phrase (e.g., having mastered the test), ask yourself who (or what) the doer of the action is. The
name of that person or thing MUST IMMEDIATELY FOLLOW the phrase, right after the comma. If a sentence begins with a
descriptive phrase (e.g., a Nobel Prize winner), ask yourself who (or what) the phrase is describing. The name of that person or thing
MUST IMMEDIATELY FOLLOW the phrase, right after the comma.

TEST SAMPLE A
Before enrolling in college, Sam’s study habits were poor.

EXPLAINED
Who enrolled in college? The sentence states that Sam’s study habits enrolled in college. That statement makes no
sense. The sentence implies that Sam enrolled in college, but the subject of a modifying phrase cannot be implied.
Because the phrase before enrolling in college lacks a specific noun to modify, it is a dangling modifier. The noun it
modifies is not specifically and clearly named in the sentence.

REVISED
Before enrolling in college, Sam had poor study habits.
OR: Before Sam enrolled in college, his study habits were poor.

TEST SAMPLE B
My older brother warned me that Professor Morgan’s Spanish final would be very difficult, but having studied for
twelve hours, the test seemed easy.

EXPLAINED
Who studied for twelve hours? The sentence does not tell us. The descriptive phrase dangles. The sentence needs to be
rewritten to eliminate the dangling modifier.

REVISED
My older brother warned me that Professor Morgan’s Spanish final would be very difficult, but having studied for twelve
hours, Ithought the test was easy.

TEST SAMPLE C
An exceptionally gifted tennis player, Rafael Nadal’s loyal fans travel all over the world just to watch him play.

EXPLAINED
Who is the exceptionally gifted tennis player? Rafael Nadal. The subject should be Rafael Nadal, not his fans.

REVISED
An exceptionally gifted tennis player, Rafael Nadal has many loyal fans who travel all over the world just to watch him
play.

REVIEW QUESTIONS: MODIFIERS

Review the following sentences for modifier errors. If there is an error, revise the sentence.
1. America’s leading writer of horror fiction, Stephen King’s novels and short-story collections have sold over 350 million copies
worldwide.
2. Television stations in Los Angeles are showing video footage of the police officer subduing the bank robber with a stun gun.
3. When I was a child, my favorite restaurant was the Tiki Hut, a small diner that gave a free sundae to every child in a miniature
coconut shell.
4. One of the greatest running backs in football history, O.J. Simpson’s athletic career has been overshadowed by his legal troubles.
5. Known for his quick wit and biting satire, politicians fear their interviews with Comedy Central’s Stephen Colbert.
6. Though known primarily as an actor and a comedian, Steve Martin’s success as an author came in 2001 with the publication of his
novellaShopgirl, which has sold over 300,000 copies to date.

CHAPTER 7
ADJECTIVES AND ADVERBS

(1) ADJECTIVE-ADVERB ERRORS

     An adverb, not an adjective, should be used to modify a verb, an adjective, or another adverb.
Adjectives modify nouns (e.g., green car, quick rabbit, clever script). Adverbs modify verbs, adjectives, and other adverbs (e.g.,
drivecarefully, very smart student, run especially quickly). Adjectives should not be used to modify verbs, adverbs, or other
adjectives. This error is frequently tested in the identifying-sentence-errors section.

TEST SAMPLE A
Political ads often contain inaccurate and misleading claims, but many voters take these claims serious.

EXPLAINED
This sentence incorrectly uses serious, an adjective, to modify take, a verb. How are the claims taken? Seriously, not
serious.

REVISED
Political ads often contain inaccurate and misleading claims, but many voters take these claims seriously.

TEST SAMPLE B
Under federal law, heroin dealers are treated harsher than marijuana dealers.

EXPLAINED
Harsher is the comparative form of the adjective harsh (harsh, harsher, harshest). The adjective harsher should not be
used to modify the verb treat. Treat should be modified by the comparative form of the adverb harshly (harshly, more
harshly, most harshly).

REVISED
Under federal law, heroin dealers are treated more harshly than marijuana dealers.

TEST SAMPLE C
Many celebrities, including Clark Gable, Humphrey Bogart, and Michael Jackson, are buried in the heavy guarded Forest
Lawn Memorial Park in Glendale, California.

EXPLAINED
The adjective heavy should not be used to modify the adjective guarded. How is the memorial park
guarded? Heavily, not heavy.

REVISED
Many celebrities, including Clark Gable, Humphrey Bogart, and Michael Jackson, are buried in the heavily guarded Forest
Lawn Memorial Park in Glendale, California.

(2) ABSOLUTE ADJECTIVES

 Absolute adjectives do not have degrees of comparison. For example, something is either unique or it is not. It cannot be
more unique, most unique, very unique, or somewhat unique.
Some adjectives describe qualities that cannot be compared; something either has the quality or it doesn’t. These adjectives are
called absolute adjectives, and they do not have degrees of comparison. One such adjective is  unique, which is the most commonly
tested absolute adjective. Other absolute adjectives are listed here:
absent infinite
absolute optimal
circular parallel
dead perfect
essential square
eternal ultimate
fatal unanimous
flat universal
immortal  

TEST SAMPLE
Rapper Fat Joe has many unusual pieces of jewelry, but his enormous Terror Squad necklace is the most unique.

EXPLAINED
Unique means one of a kind. Something is either one of a kind or not one of a kind. The necklace cannot be the most
unique.

REVISED
Rapper Fat Joe has many unusual pieces of jewelry, but his enormous Terror Squad necklace is unique.

NOTE: NEARLY PERFECT IS FINE, VERY PERFECT IS NOT

Some absolute adjectives may be preceded by nearly or almost.


Alicia Sacramone’s floor exercise routine was nearly perfect.
Mr. O’Malley has been disabled since 1995, when he suffered a nearly fatal crane accident.
It is never proper, however, to use words such as more, most, very, a little, or pretty with absolute adjectives. Be on alert for
sentences on the test that include phrases such as most perfect, more optimal, pretty circular, or very square.
REVIEW QUESTIONS: ADJECTIVES AND ADVERBS

Review the following questions for errors using adjectives and adverbs. If the sentence has an error, correct it.
1. Researchers are trying to identify the most optimal drug for the treatment of asthma.
2. At the assembly, Principal Frick warned students that anyone caught cheating would be disciplined severe.
3. Cara’s old truck runs more smooth than Cody’s new sports car.
4. Judges impose harsh sentences on defendants found guilty of treating animals cruel.
5. Although the police caught the bank robbers quickly, it took three years for the police to find the careful hidden loot.

CHAPTER 8
DICTION AND IDIOM

(1) DICTION: WRONG WORD

 Beware of words that sound alike but are spelled differently and have different meanings.
On occasion, the SAT question writers test your ability to spot a word that does not belong in the sentence. Often the word sounds
right or is close to the right word but has the wrong meaning for the sentence.

TEST SAMPLE A
Senator Upright always votes against restrictive election laws because she believes they violate democratic principals.

EXPLAINED
Principals is the wrong word. Principal, when used as a noun, means the head official, such as the principal of a school.
The correct word is principles. Principle is a noun that means rule or doctrine.

REVISED
Senator Upright always votes against restrictive election laws because she believes they violate democratic principles.

TEST SAMPLE B
Dr. Quackenbush is studying the affects of Vitamin E on the severity and duration of the common cold.

EXPLAINED
Affects is the wrong word. The correct word is effects. Affect is a verb that means to influence. Effect is usually used as a
noun that means a result.

REVISED
Dr. Quackenbush is studying the effects of Vitamin E on the severity and duration of the common cold.
For additional examples of confusing word pairs, see Appendix B.

(2) IDIOM: PREPOSITIONAL IDIOMS

     Choose the preposition that fits the idiom.


Idioms are words that go together to form expressions. The most frequently tested idioms are idioms ending in prepositions, such
as desirous ofor different from. Below are listed some of the prepositional idioms that have been tested on the SAT.
capable of ought to
credited with renowned for
encouraged by succeeded in
expel into subject to
mistrust of tendency to
PREPOSITIONAL IDIOMS ARE ALWAYS TESTED ON THE SAT, SO MAKE SURE YOU REVIEW THE LIST OF PREPOSITIONAL IDIOMS IN
APPENDIX C.

TEST SAMPLE A
The tax bill signed by the President is different than the tax bill originally proposed by Senator Smith.

EXPLAINED
The correct idiom is different from.

REVISED
The tax bill signed by the President is different from the tax bill originally proposed by Senator Smith.

TEST SAMPLE B
Last year, American manufacturers recalled over thirty models of toy trucks assembled in China because the toys did
not conform with the United States government’s safety standards.

EXPLAINED
Conform with is incorrect. The correct idiom is conform to.

REVISED
Last year, American manufacturers recalled over thirty models of toy trucks assembled in China because the toys did
not conform tothe United States government’s safety standards.

(3) IDIOM: MATCHING WORD PAIRS

     Word pairs (such as neither . . . nor, either . . . or, and both . . . and) must be matched appropriately.
Certain words, especially words of contrast and comparison, appear in pairs.
as . . . as
neither . . . nor
either . . . or
not only . . . but also
both . . . and
whether . . . or
When a word such as neither, both, or whether is used in a sentence as part of a word pair, its partner must also be present in the
sentence, and no substitutes are permitted. A common error on the SAT is the mismatching of word pairs, such as  neither . . .
or, or both . . . as well as.These pairings are grammatically incorrect. Neither must be paired with nor, and both must be paired with
and.

TEST SAMPLE A
Neither the economic stimulus package, which reduces the capital gains tax and lowers income tax rates, or the jobs
bill, which funds retraining programs and extends unemployment benefits, is expected to boost the economy quickly.

EXPLAINED
The correct conjunction pair is neither . . . nor, not neither . . . or.

REVISED
Neither the economic stimulus package, which reduces the capital gains tax and lowers income tax rates, nor the jobs
bill, which funds retraining programs and extends unemployment benefits, is expected to boost the economy quickly.

TEST SAMPLE B
Condoleezza Rice is both a foreign policy expert as well as an accomplished pianist.

EXPLAINED
The correct conjunction pair is both . . . and. This error can be tough to spot because writers and speakers often
substitute phrases such as as well as or in addition to for the word and. When both is used, however, only and is
acceptable.

REVISED
Condoleezza Rice is both a foreign policy expert and an accomplished pianist.

(4) IDIOM: BETWEEN THIS AND THAT

   The correct expression is between this and that, not between this or that.


SAT writing questions frequently test your ability to spot incorrectly worded expressions. One such expression is between [this] or
[that]. The correct expression is between [this] and [that]. Though between [this] and [that] is one of the most frequently tested
expressions, it is not the only expression tested on the SAT. You should be familiar with several other expressions:

prefer [this] to [that] NOT prefer [this] and not [that]

as [happy] as NOT as [happy] than

more [this] than [that] NOT more [this] and not [that]

[this] is the same as [that] NOT [this] is the same of [that]

from [X] to [Y] NOT from [X] and [Y]

TEST SAMPLE
Today an exceptionally talented college athlete must often make a choice between staying in college to earn a degree
or quitting college to sign with a professional team.

EXPLAINED
The correct expression is between [this] and [that], not between [this] or [that].

REVISED
Today an exceptionally talented college athlete must often make a choice between staying in college to earn a
degree and quitting college to sign with a professional team.

(5) DICTION: NUMBER VS. AMOUNT

   Number should be used to describe things that can be counted in units. Amount should be used to describe things
that can be measured but not counted.
Use amount for things that can be measured but not counted (e.g., sugar, sand, milk, work, stress) and number for things that can be
counted in units (e.g., dogs, reports, people, slices of cheese, quarts of milk).

TEST SAMPLE A
The amount of potato salad was insufficient to feed the hungry crowd, but the amount of fried chicken legs was
adequate.

EXPLAINED
Potato salad can be measured but not counted, so the use of amount is appropriate. In contrast, fried chicken legs are
countable.Number should be used.

REVISED
The amount of potato salad was insufficient to feed the hungry crowd, but the number of fried chicken legs was
adequate.

TEST SAMPLE B
Bill’s neighbor often complains about the amount of dismantled cars on Bill’s front lawn, as well as the amount of
motor oil that flows from Bill’s driveway to the bike path.

EXPLAINED
Oil can be measured but not counted, so the use of amount is appropriate. In contrast, cars are
countable. Number should be used for the noun cars.

REVISED
Bill’s neighbor often complains about the number of dismantled cars on Bill’s front lawn, as well as the amount of motor
oil that flows from Bill’s driveway to the bike path.

(6) DICTION: FEWER VS. LESS; MANY VS. MUCH

   Fewer and many should be used for things that can be counted in units. Less and much should be used for things
that can be measured but not counted.

TEST SAMPLE A
Senior year is usually more enjoyable than junior year because seniors have less stress and take less classes than
juniors do.

EXPLAINED
Classes can be counted. Use fewer. Stress, however, can be measured (high or low) but not counted. Use less.

REVISED
Senior year is usually more enjoyable than junior year because seniors have less stress and take fewer classes than
juniors do.

TEST SAMPLE B
The Attorney General of New York alleges that Merrill Lynch distributed large bonuses to much of its employees just
before the company was sold to Bank of America.

EXPLAINED
Employees are countable. Use many, not much.

REVISED
The Attorney General of New York alleges that Merrill Lynch distributed large bonuses to many of its employees just
before the company was sold to Bank of America.

(7) DICTION: BETWEEN VS. AMONG

   Use between to show the relationship between two things. Use among for three or more things.

TEST SAMPLE
Thanksgiving is usually a joyous time for everyone in our family, but this year the constant bickering among Santino and
Fredo, culminating in a food fight at the dinner table, ruined everyone’s day.

EXPLAINED
Only two people bickered. Between, not among, should be used.

REVISED
Thanksgiving is usually a joyous time for everyone in our family, but this year the constant bickering  between Santino
and Fredo, culminating in a food fight at the dinner table, ruined everyone’s day.
For additional examples of diction errors, see Appendix D.

REVIEW QUESTIONS: DICTION AND IDIOM


Review the following sentences for diction and idiom errors. If the sentence has an error, correct it.
1. If I had followed my mother’s advise, I never would have quit high school and moved to Hollywood to pursue an acting career.
2. Many dog owners argue that breed bans discriminate about dogs that are not dangerous.
3. The plot of the film Gone Baby Gone is different than the plot of the novel Gone Baby Gone by Dennis Lehane.
4. Carmen was so preoccupied in her science project that she forgot to do any other homework.
5. I chose the frozen yogurt because it has less calories and less fat than ice cream.
6. The latest government report shows that the amount of unemployed postal workers has doubled since last year.
7. Jessica has neither the athletic ability to succeed in sports or the motivation to succeed in school.
8. Mike Myers is both a brilliant physical comedian as well as a successful screenwriter.
9. Spencer prefers hiking in the Rockies and not hiking in the Cascades.
10. Japanese automobile manufacturers use the same equipment of American automobile manufacturers.

CHAPTER 9
LOGIC

(1) NOUN-NOUN AGREEMENT: “JACK AND JILL WANT TO BE TEACHERS”

     Sometimes logic requires that plural subjects be followed by plural nouns in the predicate.
Sentences have two basic parts: the subject and the predicate. The subject is who or what the sentence is about. The predicate is the
part of the sentence that tells something about the subject. The predicate includes the verb. Sometimes a plural subject or multiple
subjects are followed by singular nouns in the predicate.

EXAMPLE
A High school seniors must pass the school district’s competency test in order to graduate.
This sentence has a plural subject, seniors, but the noun test in the predicate is singular. The use of the singular
noun test is appropriate because all seniors take the same test.

EXAMPLE B
Heidi and Spencer showed Audrina their wedding album.
The use of the singular album is correct. Heidi and Spencer have one wedding album that they jointly own.

EXAMPLE C
Lil Wayne shocked his fans when he announced that he and Lil Jon would be touring the United States as a country
music duo.
In this sentence, Lil Wayne and Lil Jon together are one duo, so the singular duo is correct.
In other sentences, however, logic requires that plural subjects be matched with plural nouns in the predicate. Consider the
following examples:

WRONG
High school seniors must park their car in the rear parking lot.

EXPLAINED
The use of the singular noun car creates a logic problem. The sentence suggests that the seniors have one car that they
all share. In fact, each senior has his or her own car. The plural cars must be used.

REVISED
High school seniors must park their cars in the rear parking lot.

WRONG
Heidi and Spencer showed Audrina their high school diploma.

EXPLAINED
Heidi and Spencer do not share one diploma. There are two diplomas.

REVISED
Heidi and Spencer showed Audrina their high school diplomas.

WRONG
Lil Wayne and Lil Jon will be shaving their head for a charity event in Atlanta next week.

EXPLAINED
Lil Wayne and Lil Jon do not share one head. The plural heads should be used.

REVISED
Lil Wayne and Lil Jon will be shaving their heads for a charity event in Atlanta next week.
Although SAT question writers test noun-noun agreement in several ways, the most common pattern is  X and Y want to be a
[singer]: Jack and Jeff want to major in music and become a singer. The error in this sentence is the singular singer. Jack and Jeff are
not one singer. The singular singer should be the plural singers. Another common way to test noun-noun agreement is to describe a
singular noun in plural terms or a plural noun in singular terms. Consider the following sentence:
The National Wildlife Foundation awarded Carmen a grant to study the behavior of the kinkajou, arboreal mammals
found in South and Central America.
The noun kinkajou is singular, but it is defined as mammals, which is plural. The sentence should be revised so the two nouns match
in number:
The National Wildlife Foundation awarded Carmen a grant to study the behavior of the kinkajou, an
arboreal mammal found in South and Central America.
OR: The National Wildlife Foundation awarded Carmen a grant to study the behavior
of kinkajous, arboreal mammals found in South and Central America.

TEST SAMPLE A
After watching every episode of Dancing with the Stars, Martina and Andre decided that they want to study ballet and
become a professional dancer.

EXPLAINED
Can Martina and Andre be one dancer? No. They want to be dancers.

REVISED
After watching every episode of Dancing with the Stars, Martina and Andre decided that they want to study ballet and
become professional dancers.

TEST SAMPLE B
Lemurs, a primate found on the island of Madagascar, live in groups that are matriarchal.

EXPLAINED
This sentence follows a different pattern. The plural subject lemurs is identified in the sentence as a primate, which is
singular.

REVISED
Lemurs, primates found on the island of Madagascar, live in groups that are matriarchal.
OR: The lemur, a primate found on the island of Madagascar, lives in groups that are matriarchal.

TEST SAMPLE C
In the sixth and seventh centuries, Irish monks turned monasteries into centers of learning and devoted their life to
copying manuscripts, including the famous Book of Kells.
EXPLAINED
The error is in the clause Irish monks . . . devoted their life. The Irish monks did not share one life. The singular life should
be changed to the plural lives.

REVISED
In the sixth and seventh centuries, Irish monks turned monasteries into centers of learning and devoted their  lives to
copying manuscripts, including the famous Book of Kells.

(2) ILLOGICAL SENTENCES

 The subject of the sentence must be logically connected to the action in the sentence.
Grammar experts call this error faulty predication, which means that the subject and the predicate do not make sense together. The
key to spotting these errors is to find the subject. If the subject and the action of the verb do not make sense together, the sentence
needs to be revised.

WRONG
The purpose of the First Amendment was adopted to protect free-speech rights.

EXPLAINED
The subject is purpose. The purpose . . . was adopted makes no sense. The First Amendment was adopted, not the
purpose.

REVISED
The purpose of the First Amendment is to protect free-speech rights.
OR: The First Amendment was adopted to protect free-speech rights.

WRONG
The debate about the constitutionality of modern gun laws remains a subject of debate among legislators.

EXPLAINED
The subject is debate. The debate . . . remains a subject of debate makes no sense. What remains a subject of debate is
the constitutionality of modern gun laws, not the debate.

REVISED
The constitutionality of modern gun laws remains a subject of debate among legislators.
These errors are not usually tested directly; rather, they are tested indirectly as answer choices that should be eliminated.

TEST SAMPLE
Both the lab instructions as well as the textbook reminds students that water boils at 212 degrees Fahrenheit.
A. Both the lab instructions as well as the textbook reminds students that
B. Both the lab instructions and the textbook reminds students that
C. Both the lab instructions and the textbook remind students that the temperature of
D. Both the lab instructions and the textbook remind students that
E. The lab instructions and the textbook both reminds students that
The original sentence has two primary errors. First, it has the mismatched word pair both . . . as well as. The correct word pair
is both . . . and.Second, the sentence has an error in subject-verb agreement. Both X and Y requires a plural verb. Choices A, B, and E
fail to correct both errors. Choices C and D correct the original errors, but choice C is illogically constructed. The
subject temperature is mismatched with the verb boils. What boils is water, not temperature. The correct answer is D.

(3) THE REASON IS BECAUSE


   The expression the reason is because is redundant and ungrammatical. Use either because OR the reason is
that, but not both.
The expression the reason is because is common in speech but incorrect in formal writing. Use either because OR the reason is
that, but not both.

TEST SAMPLE
According to the latest research, the reason birds migrate is because food in their habitat is scarce.

EXPLAINED
This sentence sounds right but has an error. The sentence uses the grammatically incorrect expression  the reason is
because.

REVISED
According to the latest research, birds migrate because food in their habitat is scarce.
OR: According to the latest research, the reason birds migrate is that the food in their habitat is scarce.

(4) IS WHEN AND IS WHERE

 Do not define or describe a subject by using is when or is where.


When refers to time, and where refers to place or location. Is when and is where should not be used in sentences that define a
subject, or in place of that, which, or in which.

WRONG
Tachycardia is a medical condition where the heart beats over 100 times a minute.

RIGHT
Tachycardia is a medical condition in which the heart beats over 100 times a minute.

WRONG
A sabbatical is when an employee takes an extended leave of absence from work.

RIGHT
A sabbatical is an extended leave of absence from work.

(5) BETTER THAN ANY OTHER

   In a sentence that compares a person or thing in a category to others in the same category, follow the pattern,  My
dog is smarter than any other dog, NOT My dog is smarter than any dog.

TEST SAMPLE
Michael Phelps is faster than any swimmer who competed in the Beijing Olympics.

EXPLAINED
This sentence has a logic error. Michael Phelps himself is a swimmer who competed in the Beijing Olympics. He cannot
be faster than himself.

REVISED
Michael Phelps is faster than any other swimmer who competed in the Beijing Olympics.

REVIEW QUESTIONS: LOGIC

Review the following sentences for logic errors. If a sentence has an error, correct it.
1. A decision by the Supreme Court on the constitutionality of life sentences for juveniles will be decided this term.
2. The reason Harold lost his job at Subway is because he threw tomato slices at the manager.
3. Kangaroos, a marsupial endemic to Australia, are the only large animals that use hopping as their primary means of locomotion.
4. Janice and her sister Charlotte want to study psychology at the University of Michigan and become a social worker.
5. A plea bargain is where the defendant agrees to plead guilty to a less serious charge in exchange for a shorter sentence.
6. The jaguar is stronger and faster than any big cat.

CHAPTER 10
SENTENCE CONSTRUCTION PROBLEMS

Errors in sentence structure are always tested in the improving-sentences or improving-paragraphs section. The three types of
construction errors are run-ons, fragments, and comma splices.

(1) RUN-ONS

 Two sentences cannot be connected without punctuation.


A run-on sentence is two or more sentences strung together without punctuation.

TEST SAMPLE
Barbara Jean stayed at the party until midnight she was too tired to go to work the next day.

EXPLAINED
This sentence is two sentences strung together without punctuation or a connector. There are four ways to correct a
run-on sentence, but usually, the correct answer on the test is the choice that inserts a comma followed by a connector
(such as but, and, or so) or a semicolon followed by a transition word (such as therefore or consequently) and a comma.

REVISED
(1) INSERT COMMA + CONNECTOR:
Barbara Jean stayed at the party until midnight, so she was too tired to go to work the next day.
(2) INSERT SEMICOLON:
Barbara Jean stayed at the party until midnight; she was too tired to go to work the next day.
(3) INSERT A SEMICOLON FOLLOWED BY A TRANSITION WORD + COMMA:
Barbara Jean stayed at the party until midnight; consequently, she was too tired to go to work the next day.
(4) SEPARATE INTO TWO COMPLETE SENTENCES:
Barbara Jean stayed at the party until midnight. She was too tired to go to work the next day.
NOTE ABOUT TRANSITION WORDS
“Transition word” is not a grammar term but rather a simple way to refer to what are called conjunctive adverbs. Conjunctive
adverbs are words and phrases such as thus, therefore, as a result, likewise, in contrast, however, nevertheless, consequently,
otherwise, moreover, accordingly, and furthermore. Remember that if one of these words is used to join two sentences, a semicolon
needs to be placed before the word and a comma needs to follow the word:
Omar needs to pass chemistry this term; otherwise, he cannot graduate.
The budget for the school district has been cut in half; as a result, no new teachers will be hired this year.

(2) FRAGMENTS

     A sentence must have a subject and a verb, and express a complete thought. When two sentences are
connected by a conjunction, each sentence must be complete.
If a sentence lacks a subject or a verb, or fails to express a complete thought, it is a fragment. The SAT always tests your ability to
spot and correct fragments, and fragment questions usually follow one of three patterns: (1) the missing verb; (2) the fake verb; or
(3) the faulty clause.
THE MISSING VERB. SAT question writers often create fragments by omitting the verb.

TEST SAMPLE A
Stephen King, noted writer of horror fiction and fantasy novels, winner of the 2003 Medal for Distinguished Contribution to
American Letters.

EXPLAINED
This sentence has a subject, Stephen King, but no verb.

REVISED
Stephen King, noted writer of horror fiction and fantasy novels, was the winner of the 2003 Medal for Distinguished
Contribution to American Letters.
OR: Stephen King, noted writer of horror fiction and fantasy novels, won the 2003 Medal for Distinguished Contribution to
American Letters.
THE FAKE VERB. SAT question writers often substitute modifying phrases for verbs in an effort to make a fragment look as if it has
both a subject and a verb. These modifying phrases include words that look like verbs but do not act like verbs. Examples of
modifying phrases (in italics) are “stars shining at night,” “children screaming on airplanes,” “freight carried by rail,” and
“wires attached to a converter box.”These phrases describe the nouns that precede them.

TEST SAMPLE B
Every year a dedicated group of volunteers in the Alaskan city of North Pole answering thousands of letters mailed to Santa
Claus.

EXPLAINED
This group of words is a fragment. The subject group (of volunteers) needs a verb. Answering looks like a verb, but it is
not. Answering thousands of letters is a phrase that describes the noun volunteers: volunteers answering thousands of
letters. Similarly, the word mailedlooks like a verb but isn’t. The phrase mailed to Santa Claus describes the noun letters:
letters mailed to Santa Claus. In this test sample,answering must be changed to the verb answers.

REVISED
Every year a dedicated group of volunteers in the Alaskan city of North Pole answers thousands of letters mailed to Santa
Claus.
THE FAULTY CLAUSE. Remember that sentences often have more than one clause. Sometimes two clauses are connected by a
conjunction such as and, but, or so (e.g., Omar will bake the cookies, and I will prepare the salad). Other times a dependent clause is
connected to an independent clause (e.g., Bob will be at the meeting by 9:30 a.m. unless his flight is delayed). In a sentence with
more than one clause, each clause must be complete. In some SAT questions, one clause is complete, but the other is a fragment.

TEST SAMPLE C
Whereas Mike, who has visited four continents, Vinnie has never traveled outside the United States.

EXPLAINED
The subordinating conjunction whereas should introduce a dependent clause, and that dependent clause should be
connected to an independent clause. (e.g., Whereas John enjoys classical music, his brother Jimmy listens only to
rap. OR: John enjoys classical music,whereas his brother Jimmy listens only to rap.). In this test sample, the dependent clause
has a subject (Mike) but no verb. The relative clause who has visited four continents does not provide the verb. Remember
that relative clauses are add-ons: they are added to sentences or clauses that are otherwise complete. You should be able to
delete the relative clause and still have a main clause that is complete. If you strike out the relative clause in this test sample,
you can see that the dependent clause is missing a verb: Whereas Mike, who has visited four continents, Vinnie has never
traveled outside the United States.

REVISED
Whereas Mike has visited four continents, Vinnie has never traveled outside the United States.
OR: Unlike Mike, who has visited four continents, Vinnie has never traveled outside the United States.
TEST TIP: LOOK FOR WORDS SUCH AS WHEREAS AND BECAUSE
Many of the fragments on the SAT contain subordinating conjunctions such as whereas, unless, even though, or because. These
subordinating conjunctions should introduce dependent clauses that have both a subject and a verb.
WRONG Whereas Snooki, who is obnoxious and has few friends, Sammi is personable and has many friends.

RIGHT Whereas Snooki is obnoxious and has few friends, Sammi is personable and has many friends.
OR: Unlike Snooki, who is obnoxious and has few friends, Sammi is personable and has many friends.

WRONG Snooki has few friends because her offensive personality.

RIGHT Snooki has few friends because she has an offensive personality.


OR: Snooki has few friends because of her offensive personality.

Note the difference between because and because of. Because is a conjunction that must be followed by a clause with a subject and
a verb. Because of is a (compound) preposition that can be followed by a noun or pronoun.

(3) THE COMMA SPLICE

     Two sentences cannot be connected by a comma alone.


A comma splice is two complete sentences connected by only a comma. Two sentences CANNOT be connected by a comma alone.
This rule is always tested on the SAT.

EXAMPLE
The principal wants to adopt a strict dress code for teachers, most of the teachers oppose the change.

EXPLAINED
This sentence is a comma splice: it is two sentences joined by a comma alone.

REVISED
In general, there are five ways to correct a comma splice.
(1) ADD CONNECTOR AFTER THE COMMA:
The principal wants to adopt a strict dress code for teachers, but most of the teachers oppose the change.
(2) CHANGE COMMA TO SEMICOLON:
The principal wants to adopt a strict dress code for teachers; most of the teachers oppose the change.
(3) INSERT SEMICOLON + TRANSITION WORD BEFORE THE COMMA:
The principal wants to adopt a strict dress code for teachers; however, most of the teachers oppose the change.
(4) SEPARATE INTO TWO COMPLETE SENTENCES:
The principal wants to adopt a strict dress code for teachers. Most of the teachers oppose the change.
(5) CHANGE ONE OF THE SENTENCES TO A PHRASE OR A RELATIVE CLAUSE:
The principal wants to adopt a strict dress code for teachers, most of whom oppose the change.
TEST TIP: MOST COMMA SPLICES ON THE SAT CAN BE CORRECTED BY CHANGING ONE OF THE SENTENCES TO A PHRASE OR A
RELATIVE CLAUSE
SAT question writers always test comma splices. Comma splices are tested in all three multiple-choice sections: Identifying Sentence
Errors, Improving Sentences, and Improving Paragraphs. In almost every comma-splice question on the SAT, there is no need to
connect two complete sentences, and the comma splice can be corrected by changing one of the sentences to a phrase or a relative
clause. Consider the following example:
This sentence is a comma splice: it is two sentences connected by a comma alone. You need to identify the part of the sentence that
makes the sentence a comma splice and needs to be revised. In this example, the error occurs at A. You can change the first
sentence in the comma splice to a phrase by deleting the words It is.
The world’s largest reef system, the Great Barrier Reef is comprised of more than 3000 coral reefs and 600 islands.

TEST SAMPLE A
The first dinosaur eggs were found in the Gobi Desert, it is the largest desert region in Asia.

EXPLAINED
This sentence is a comma splice: two complete sentences are joined by a comma alone. Though you could correct this
comma splice by changing the comma to a semicolon, there is no need to connect two complete sentences here. The
pronoun it is unnecessary. The best way to revise this sentence is to change the second sentence to a descriptive phrase.

REVISED
The first dinosaur eggs were found in the Gobi Desert, the largest desert region in Asia.

TEST SAMPLE B
In 1838 Frederick Douglass escaped to the free state of Massachusetts, there he made speeches against slavery and
wrote his autobiography.

EXPLAINED
Again, there is no need to connect two complete sentences here. The comma splice can be corrected by changing the
second sentence to a relative clause.

REVISED
In 1838 Frederick Douglass escaped to the free state of Massachusetts, where he made speeches against slavery and
wrote his autobiography.

TEST SAMPLE C
Wendy Moffat’s new biography of novelist E. M. Forster draws on Forster’s extensive collection of personal letters,
most of them were kept private until 2008.

EXPLAINED
This comma splice can be corrected by changing the second sentence to a modifying clause. Look out for this pattern on
the SAT. Many times the first sentence ends with a noun, and the second sentence starts with the same noun or a
pronoun that refers to that noun. In this example, the first sentence ends with the noun  letters, and the second sentence
starts with most of [the letters]. In such sentences, the comma splice can usually be corrected by changing the second
sentence to a relative clause. Here, you can changemost of them to most of which.

REVISED
Wendy Moffat’s new biography of novelist E. M. Forster draws on Forster’s extensive collection of personal letters, most
of whichwere kept private until 2008.
CHAPTER 11
CONNECTING AND COORDINATING PARTS OF A SENTENCE

(1) AND OR BUT?

     Two clauses joined by and should sometimes be joined by but or some other connector.
The connector and should be used to connect words or phrases that are equivalent.
I will bring lasagna, Jim will bring a salad, and Omar will bring cupcakes.
And is not a universal connector, however. And should not be used if another connector can more precisely define the relationship
between the two clauses. In sentences that show contrast, for example, but is a better choice than and. Similarly, in sentences that
show a cause-and-effect relationship between two events, the connector so or the conjunction because is a better choice than and.
TEST SAMPLE A
The alligator has a broad snout, and the crocodile has a thin snout.

EXPLAINED
This sentence contrasts the characteristics of two similar animals, but the connector and does not show contrast.

REVISED
The alligator has a broad snout, but the crocodile has a thin snout.

TEST SAMPLE B
The circumstances of the singer’s death were suspicious, and the police decided to investigate the death as a possible
homicide.

EXPLAINED
In this sentence, there is a cause-and-effect relationship between the two clauses. The suspicious circumstances of the
singer’s death caused the police to investigate.

REVISED
The circumstances of the singer’s death were suspicious, so the police decided to investigate the death as a possible
homicide.

TEST SAMPLE C
Barney ordered the extra-large pizza, and he is hungry.

EXPLAINED
This sentence has a cause-and-effect relationship. Barney ordered the large pizza because he is hungry.

REVISED
Barney ordered the extra large pizza because he is hungry.
Here’s an example of how this error is likely to appear on the test:

The error is C. This sentence compares two types of cheese. To show contrast, the connector  but should be used: Parmesan cheese
is made from cow’s milk, but feta cheese is made from sheep’s milk.

(2) TRANSITION AND COORDINATION

 The connector must fit the context.


Some writing questions test your ability to choose the connector or transition word that fits the context of the sentence. The
connectors and transition words most likely to appear on the test fall into five general categories: comparison, contrast, cause,
effect, or addition.
COMPARISON: as, like, likewise, similarly
CONTRAST: although, but, even though, however, instead, nevertheless, though, yet
CAUSE: because, if, since
EFFECT: accordingly, as a result, consequently, for, so, therefore, thus
ADDITION: also, and, besides, furthermore, moreover

TEST SAMPLE
Lacto-ovo vegetarianism allows for the consumption of eggs and dairy products; therefore, veganism forbids the
consumption of all animal products, including eggs, milk, and honey.

EXPLAINED
Therefore is the wrong connector for this sentence. Therefore is more appropriately used in a sentence that shows cause
and effect:Bob is a vegan; therefore, we made him a soy burger. The sample sentence is not a cause-and-effect sentence;
rather, it contrasts two types of vegetarianism.

REVISED
Lacto-ovo vegetarianism allows for the consumption of eggs and dairy products,  but veganism forbids the consumption
of all animal products, including eggs, milk, and honey.
OR: Whereas lacto-ovo vegetarianism allows for the consumption of eggs and dairy products, veganism forbids the
consumption of all animal products, including eggs, milk, and honey.

(3) PUNCTUATION: SEMICOLONS AND COLONS

     Use commas, semicolons, and colons appropriately.


The SAT always tests your ability to spot and correct comma splices and fragments. Sometimes the error is tested directly: The
question itself is a comma splice or fragment and you have to pick the answer that corrects the error. Other times, the error is tested
indirectly in answer choices: Two choices will be identical except for punctuation. One choice creates a comma splice or fragment,
but the other does not. You need to know how to use commas, semicolons, and colons correctly so you can eliminate these faulty
answers quickly.

SEMICOLONS
SEMICOLONS are used to connect two complete sentences.
Carmelo passed chemistry; now he has enough credits to graduate.
Sam is very close to his mother; he calls her every day.
Sometimes a semicolon is followed by a transition word (e.g., however, therefore, besides, still, consequently, otherwise, thus) and a
comma.
Marlena’s lease has expired; therefore, she will be moving to a new apartment.
The number of women admitted to graduate business schools is increasing; however, the number of women in executive
positions at major corporations is decreasing.
DO NOT USE A SEMICOLON WITH A CONJUNCTION SUCH AS AND, OR, BUT, OR FOR. USE A COMMA.
WRONG My brother plays football; but I play soccer.
RIGHT My brother plays football, but I play soccer.
WRONG I will bake the brownies for the party; and you can bake the cookies.
RIGHT I will bake the brownies for the party, and you can bake the cookies.

COLONS
A colon, like a semicolon, may be used to connect two sentences. A colon should be used to connect two sentences only when the
second sentence explains or illustrates the first sentence.
Justin is an animal lover: he owns two dogs, a cat, a guinea pig, and an iguana.
The mayor has proposed a new solution to the downtown parking problem: he wants to convert the downtown library to
a parking garage.
A colon may also be used to introduce a list or series.
Pluto has three moons: Charon, Nix, and Hydra.
Students registered for the test are advised to bring the following: two pencils, a soft eraser, a calculator, and a watch.
Finally, a colon may be used to introduce a clarification or example.
Shakira gave the crowd what it wanted: a spirited performance of “Hips Don’t Lie.”
Great musicians, athletes, and actors share one common trait: the desire to succeed.

(4) PUNCTUATION: COMMA OR NO COMMA?

   Nonessential information should be set off with commas. Essential information should not be set off with commas.
This rule shows up on the SAT in two ways:
(1) in that-vs.-which questions and
(2) in questions with appositives.
(1) That vs. which. Remember that the relative pronoun that is used for essential information and the relative pronoun which is used
for nonessential information. For that reason, a which clause is set off by commas but a that clause is not.
WRONG Witnesses described the getaway vehicle as a minivan, that has bright red racing stripes and oversized tires.
RIGHT Witnesses described the getaway vehicle as a minivan that has bright red racing stripes and oversized tires.
The relative pronoun that is required because the relative clause is essential to the meaning of the sentence. The clause identifies
the kind of minivan that the witnesses saw: one with red racing stripes and oversized tires. Because the clause is essential, the
relative pronoun that is required, and the clause should not set off by a comma.
WRONG Snooki used her father’s credit card to buy a new bicycle which is blue.
RIGHT Snooki used her father’s credit card to buy a new bicycle, which is blue.
In this sentence the relative clause which is blue adds information, but that information is not essential to the meaning of
the sentence. The point of the sentence is that Snooki used her father’s credit card to buy a bicycle. The which clause
should be set off by a comma.
(2) Appositives. An appositive is a noun or noun phrase placed next to another noun or noun phrase. The appositive explains or
renames the noun to which it attaches (e.g., Chuckles, a local clown; noted author Stephen King; rapper Fat Joe). Appositives are
either nonessential or essential. Nonessential appositives are set off by commas, whereas essential appositives are not. An essential
appositive is restrictive: it narrows or limits the phrase that precedes it and answers the question, “Which one?” (e.g.,  actor and
comedian Dane Cook; author James Joyce; my friend Sal). A nonessential appositive is merely an add-on: it renames, explains, or
describes the noun next to it (e.g., my twin sister, Jasmine; Bill Clinton, the forty-second president; Gretchen Carlson, a former Miss
America). Note that if you have only one brother, the name of your brother is nonessential (my brother, Bill), but if you have more
than one brother, the name of the brother is essential (my brother Bill). Bill answers the question, “Which one?” For purposes of the
SAT, look for nouns or noun phrases placed next to each other, and don’t worry about which of the two is the appositive. All you
need to be able to do is recognize the patterns shown in the following examples (appositives in italics):

NO COMMA:
Louisiana native Lil Wayne will be performing in New Orleans tonight.
[The appositive is essential. It answers the question, “Which Louisiana native?” The noun Lil Wayne identifies the specific
Louisiana native who will be performing.]

COMMAS:
Lil Wayne, a Louisiana native, will be performing in New Orleans tonight.
[The noun phrase a Louisiana native adds information, but that information is not essential to the meaning of the
sentence. You can strike out the phrase without changing the meaning of the sentence.]

COMMA:
A Louisiana native, Lil Wayne will be performing in New Orleans tonight.
[Again, the noun phrase a Louisiana native adds information, but that information is not essential to the meaning of the
sentence. You can strike out the phrase without changing the meaning of the sentence.]
Here are a few other examples:

NO COMMA:
The Colorado band OneRepublic found fame on MySpace.

COMMAS:
OneRepublic, a Colorado band, found fame on MySpace.

COMMA:
A Colorado band, OneRepublic found fame on MySpace.

NO COMMA:
The Greek philosopher and mathematician Plato founded the first university in the Western world.
COMMAS:
Plato, a Greek philosopher and mathematician, founded the first university in the Western world.

COMMA:
A Greek philosopher and mathematician, Plato founded the first university in the Western world.

NO COMMA:
American novelist and playwright Langston Hughes left Columbia University in 1922 because of racial prejudice.

COMMAS:
Langston Hughes, an American novelist and playwright, left Columbia University in 1922 because of racial prejudice.

COMMA:
An American novelist and playwright, Langston Hughes left Columbia University in 1922 because of racial prejudice.
The questions that test this rule are usually found in the improving-sentences section of the SAT, and they follow a common pattern.

TEST SAMPLE
Clint Eastwood, veteran actor and director, as the recipient of five Academy Awards and five Golden Globe Awards.
A. Clint Eastwood, veteran actor and director, as
B. Veteran actor and director Clint Eastwood is
C. Veteran actor and director, Clint Eastwood is
D. Clint Eastwood was a veteran actor and director as
E. A veteran actor and director, it was Clint Eastwood as
This sample question is a fragment because it is missing a verb. Choices B and C both provide the missing verb, but choice C places a
comma where it doesn’t belong. The noun phrase veteran actor and director and the noun Clint Eastwood should not be separated
by a comma. The noun Clint Eastwood answers the question, “Which veteran actor and director?” The correct answer is B: Veteran
actor and director Clint Eastwood is the recipient of five Academy Awards and five Golden Globe Awards.  A couple of other sentences
would also have been correct:
A veteran actor and director, Clint Eastwood is the recipient of five Academy Awards and five Golden Globe Awards.
OR: Clint Eastwood, a veteran actor and director, is the recipient of five Academy Awards and five Golden Globe Awards.

REVIEW QUESTIONS: SENTENCE CONSTRUCTION AND COORDINATION

Revise the following sentences to correct sentence construction and coordination errors.
1. A federal judge is appointed by the President, and a state judge is elected by the voters.
2. In 1864 artist Mary Cassatt moved to Paris, there she befriended a group of independent artists later known as the Impressionists.
3. Whereas the Toyota Prius is a hybrid vehicle that runs on both electricity and gas; the Chevrolet Volt is a plug-in vehicle that runs
exclusively on electricity.
4. Janet and her brother Bruno are vegetarians: only fruits, vegetables, and whole grains.
5. The performance artist, who painted his body green and called himself a tree.
6. Jacob Lawrence was one of the most important artists of the twentieth century, he is best known for his series of paintings
depicting important moments in African-American history.
7. American Idol winner, Kelly Clarkson has released four successful albums, one of which was the multi-platinum Breakaway.
8. Whereas most airlines, which allow passengers to select seats at the time of ticket purchase, Southwest Airlines does not assign
seats to its passengers.

CHAPTER 12
CHOOSING STRONG AND CONCISE ANSWERS
The sentence- and paragraph-improvement questions ask you to select the best of five phrases or sentences. You are directed to
select the option that results in the clearest, most effective sentence that is also free of grammatical errors. In choosing the best
answer, avoid selections that are wordy, redundant, weak, or passive.

(1) WORDINESS

   Avoid answers that contain unnecessary words.


Wordiness is using more words than necessary to express a thought. An example is the phrase  on a weekly basis, which is a wordy
way of saying weekly. In the sentence- and paragraph-improvement sections, avoid choices that contain more words than necessary.
Choose answers that are both grammatically correct and concise.

TEST SAMPLE A
It is the job of the auditor to conduct an examination of accounting records and engage in the preparation of financial
reports.

EXPLAINED
This sentence has several unnecessary words and can be made more concise. It is the job of the auditor can be changed
to the auditor’s job is. To conduct an examination of can be shortened to to examine. Engage in the preparation of
financial reports can be pared down to prepare financial reports.

REVISED
The auditor’s job is to examine accounting records and prepare financial reports.

TEST SAMPLE B
In spite of the fact that its membership is shrinking, the Chester A. Arthur fan club still meets on a weekly basis.

EXPLAINED
This sentence has several unnecessary words. In spite of the fact that can be pared down to although, and the words on
a weekly basis can be pared down to weekly.

REVISED
Although its membership is shrinking, the Chester A. Arthur fan club still meets weekly.

(2) REDUNDANCY

   Avoid answers that contain redundancies.


Redundancy is the unnecessary repetition of an idea. An example of redundancy is the phrase  join together. To join means to put
together, so the addition of together is redundant. In the sentence- and paragraph-improvement sections, avoid choices that contain
redundancies. Choose answers that are grammatically correct and concise.

TEST SAMPLE
Audrina and Lauren will be collaborating together on the science project that is due in the month of May.

EXPLAINED
Collaborating together is redundant because collaborate means to work together. The month of May is also redundant.
The single word May is sufficient. The sentence can also be made more concise by dropping the superfluous that is.

REVISED
Audrina and Lauren will be collaborating on the science project due in May.
For additional examples of wordiness and redundancy, see Appendix E.

(3) PASSIVE CONSTRUCTIONS


   Avoid answers that have weak and passive constructions.
Sentences written in the active voice are constructed so that the actor does the action of the verb (e.g., Usher sang the national
anthem).Sentences written in the passive voice are constructed so that the actor receives the action of the verb (e.g., The national
anthem was sung by Usher). A passive construction is not a grammar error, but most of the time, a passive construction is weaker
and less effective than an active construction.
Passive Three coats of varnish were applied by the carpenter.
Active The carpenter applied three coats of varnish.
Passive A pod of orcas was seen by the passengers on the ferry.
Active The passengers on the ferry saw a pod of orcas.
TEST TIPS: THE IMPROVING-PARAGRAPHS SECTION
The rules covered in Chapters 10, 11, and 12 are often tested in the improving-paragraphs section. Many of the improving-
paragraphs questions require you to coordinate sentences or parts of sentences appropriately. The following examples show some
of the sentence patterns in the improving-paragraphs section.
EXAMPLE Author Willa Cather was a rebellious child. She often wore boy’s clothing to school and challenged her teachers’
A authority.
EXPLAINEDThese sentences do not contain any grammar errors, but an improving-paragraphs question may require you to combine
these sentences correctly and effectively.
REVISED A rebellious child, author Willa Cather often wore boy’s clothing to school and challenged her teachers’ authority.
EXAMPLE Author Willa Cather was a rebellious child, and she often wore boy’s clothing to school and challenged her teachers’
B authority.
EXPLAINEDIn this example, the two sentences in Example A are connected by and. Whenever you see the pattern[X] is [this], and [X]
is [that], the sentence can usually be improved by changing one of the clauses to a phrase or relative clause.
REVISED A rebellious child, author Willa Cather often wore boy’s clothing to school and challenged her teachers’ authority.
EXAMPLE In 1906 Willa Cather moved to New York. In New York she became the managing editor ofMcClure’s magazine.
C
EXPLAINEDThese sentences can be combined by changing one of them to a relative clause.
REVISED In 1906 Willa Cather moved to New York, where she became the managing editor of McClure’smagazine.
EXAMPLE Willa Cather often traveled to France, being that she enjoyed studying French culture and literature.
D
EXPLAINEDSAT question writers frequently use the unidiomatic being that in place of because or since. Whenever you see the
phrase being that in an SAT question, look for a way to revise the sentence. You can usually change being
that to because.
REVISED Willa Cather often traveled to France because she enjoyed studying French culture and literature.
EXAMPLE Cather’s novels chronicle the lives of pioneers in frontier America. My Antonia tells the story of several immigrant
E families who settle in rural Nebraska.
EXPLAINEDWhat’s missing here is a word or phrase that shows the relationship between the two sentences. The first sentence
makes a broad statement about Cather’s novels. The second sentence supports that statement with an example. Starting
the second sentence with for example clarifies the relationship between the two sentences. Many improving-paragraphs
questions require you to add words or phrases such as however, consequently, for example, nevertheless, or therefore to
show the relationship between two sentences.
REVISED Cather’s novels chronicle the lives of pioneers in frontier America. For example, My Antonia tells the story of several
immigrant families who settle in rural Nebraska.
SUPER-QUICK STUDY GUIDE: SAT WRITING ESSENTIALS

SUBJECT-VERB AGREEMENT
(1) Match singular nouns with singular verbs and plural nouns with plural verbs. Ignore distracting words and phrases between the
subject and the verb.

WRON The aroma of hamburgers, hot dogs, and ribs on the grill make my mouth water.
G

RIGHT The aroma of hamburgers, hot dogs, and ribs on the grill makes my mouth water. (subject-verb pair = aromamakes)

WRON Though the origin of the Olympic Games are unknown, historians agree that the Games were an outgrowth of the Greek
G idealization of physical fitness.
RIGHT Though the origin of the Olympic Games is unknown, historians agree that the Games were an outgrowth of the Greek
idealization of physical fitness. (subject-verb pair = origin is)

WRON The reasons for the new policy banning microwave popcorn is explained in the latest company newsletter.
G

RIGHT The reasons for the new policy banning microwave popcorn are explained in the latest company newsletter. (subject-verb
pair = reasons are)

(2) In inverted (backward) sentences, match the verb to the subject or subjects that follow the verb.

WRONG In the principal’s office is the student roster and the master class schedule.

RIGHT In the principal’s office are the student roster and the master class schedule.

Try reversing and simplifying the sentence: The roster and the schedule are in the principal’s office.
(3) Collective nouns take singular verbs.

WRONG The jury of eight men and four women are deliberating today.

RIGHT The jury of eight men and four women is deliberating today.

WRONG With no mascot, the team rely on the cheerleaders to entertain the crowd.

RIGHT With no mascot, the team relies on the cheerleaders to entertain the crowd.

(4) The number of is singular.

WRONG The number of visitors to Antarctica have quadrupled in the last decade.

RIGHT The number of visitors to Antarctica has quadrupled in the last decade.

PRONOUNS
(1) When a pronoun is paired with a noun or another pronoun, test the pronoun on its own to determine the proper case.

WRONG Impressed with our performance at the regional competition, the coach picked Carla and I for the national swim team.

Is I the correct pronoun? Test it on its own.


The coach picked I for the national swim team.
The pronoun is in the wrong case.

RIGHT Impressed with our performance at the regional competition, the coach picked Carla and me for the national swim team.

(2) Pronouns that are the objects of prepositions must be in the objective case:
to Jamal and him
from us to them
with him and her
by Sammy and him
between them and us
Pay particular attention to prepositional phrases starting with between. The common phrase between you and I is incorrect. The
correct phrase is between you and me.

WRON Though Nene has been my best friend for seven years, the recent disagreement between she and I has put a strain on our
G relationship.

Between is a preposition. The objects of a prepositional phrase must be in the objective case: between her and me.
RIGH Though Nene has been my best friend for seven years, the latest disagreement between her and me has put a strain on our
T relationship.

(3) Do not use they (or them or their) to refer to singular nouns. Use it, or he or she, to refer to singular nouns.

WRON According to the school’s honor code, a student accused of cheating or plagiarism may admit wrongdoing and accept
G suspension, or they can request a hearing to rebut the charge.

RIGHT According to the school’s honor code, a student accused of cheating or plagiarism may admit wrongdoing and accept
suspension, or he or she can request a hearing to rebut the charge.

(4) Pronouns should not switch from one to you, or you to they, or they to one.

WRONG If one plans to bring a calculator to the test, they should also bring an extra set of batteries.

RIGHT If one plans to bring a calculator to the test, one should also bring an extra set of batteries.

  OR: If you plan to bring a calculator to the test, you should also bring an extra set of batteries.

Moreover, one should not be used to refer to a noun in the sentence.

WRON For a student to be eligible for the honors program, one must have a grade point average of at least 3.75 and an SAT score
G over 2000.

RIGHT For a student to be eligible for the honors program, he or she must have a grade point average of at least 3.75 and an SAT
score over 2000.

(5) Use that when referring to singular nouns. Use those when referring to plural nouns.

WRONG The wages of American auto workers are thirty times higher than that of Chinese auto workers.

RIGHT The wages of American auto workers are thirty times higher than those of Chinese auto workers.

If you rephrase the sentence, you can see that the pronoun those is a substitute for the plural noun wages: The wages of
American auto workers are thirty times higher than the wages of Chinese auto workers.

WRONG More elongated and slender than humpbacks, fin whales are found in all seas except that of the Antarctic region.

RIGHT More elongated and slender than humpbacks, fin whales are found in all seas except those of the Antarctic region.

If you rephrase the sentence, you can see that the pronoun those is a substitute for the noun seas: More elongated and
slender than humpbacks, fin whales are found in all seas except the seas of the Antarctic region. The pronoun those is a
substitute for the nounseas, which is plural.

VERBS
(1) The verb tense must fit the time of the action.

WRON In 1767, the Parliament of Great Britain passed a series of acts known as the Townshend Acts, which will tax tea, paper, and
G other products imported into the colonies.

RIGHT In 1767, the Parliament of Great Britain passed a series of acts known as the Townshend Acts, which taxed tea, paper, and
other products imported into the colonies.

The action occurred in 1767. The past tense is required.


(2) Beware of errors in the use of irregular verbs, such as have went or have swam. The correct verbs are have gone and have
swum.

WRON By the time Chuck turned seventeen, he had climbed Mount Rainier, had drove from Maine to California, and had visited
G four continents.
RIGHT By the time Chuck turned seventeen, he had climbed Mount Rainier, had driven from Maine to California, and had visited
four continents.

To drive is an irregular verb. The present tense is drive, the past tense is drove, and the past participle is driven. The correct verb
is had driven.

PARALLEL STRUCTURE
(1) Parallelism requires that similar elements in a sentence be phrased in the same way. Use  hike, bike, and ski, not hike, bike, and
skiing.

WRONG Mike Myers is a brilliant comedian, an accomplished screenwriter, and has acted in several successful films.

RIGHT Mike Myers is a brilliant comedian, an accomplished screenwriter, and a successful actor.

All elements of the series have to be phrased in the same way. Note also that you need to include an article (a,
an, or the) for each element in the series because the article a cannot be used before the word accomplished. Mike
Myers is an accomplished screenwriter.

WRON While in Paris, Keisha and Maria climbed the Eiffel Tower, took a sightseeing cruise on the Seine, and they toured the Notre
G Dame Cathedral.

RIGHT While in Paris, Keisha and Maria climbed the Eiffel Tower, took a sightseeing cruise on the Seine, and toured the Notre Dame
Cathedral.

(2) Elements that are part of a conjunction pair must also be parallel. The correct phrase is neither hiking nor biking, not neither to
hike nor biking.

WRONG Neither the dive in the Gulf of Mexico nor hiking in the Rockies was as much fun as I thought it would be.

RIGHT Neither diving in the Gulf of Mexico nor hiking in the Rockies was as much fun as I thought it would be.

  OR: Neither the dive in the Gulf of Mexico nor the hike in the Rockies was as much fun as I thought it would be.

(3) The misplacement of a single word can create an error in parallelism. Beware of this error on the SAT. If you see a conjunction pair
such asboth . . . and or either . . . or, check for parallelism.

WRONG Soccer is popular both in Chile and Argentina.

  Is this sentence parallel? No. The first half of the conjunction pair has a preposition but the second does not.

both in Chile

and Argentina

RIGHT Soccer is popular both in Chile and in Argentina.

  OR: Soccer is popular in both Chile and Argentina.

WRONG Yoga class will be held in either the studio or community center.

  Is this sentence parallel? No. The first half of the conjunction pair has an article (the) but the second half does not.

either the studio

or community center

The appropriate article (a, an, the) must be added.

RIGHT Yoga class will be held in either the studio or the community center.


COMPARISONS
Comparisons must be logical (e.g., comparing Kanye West’s music to Eminem’s music, not Kanye West’s music to Eminem).

WRONG Unlike Greek temples, Romans added columns to their temples for decorative, not structural, purposes.

This sentence illogically compares Greek temples to Romans. The comparison should be Greek temples to Roman
temples, or Greeks to Romans, but not Greek temples to Romans.

RIGHT Unlike the Greeks, Romans added columns to their temples for decorative, not structural, purposes.

WRON Though Alaska has more land area than any other state, the population of Alaska is smaller than any other state except
G Vermont, Wyoming, and North Dakota.

This sentence illogically compares the population of Alaska with other states. The comparison should be state to state, or
population to population, but not population to state.

RIGH Though Alaska has more land area than any other state, the population of Alaska is smaller than the population of any other
T state except Vermont, Wyoming, and North Dakota.

  OR: Though Alaska has more land area than any other state, the population of Alaska is smaller than that of any other state
except Vermont, Wyoming, and North Dakota.

MODIFIERS
(1) A descriptive phrase must be placed as close as possible to the noun or pronoun that it describes.

WRONG Cabin Fever was the first feature film directed by Eli Roth, a horror film about a flesh-eating virus.

In this sentence the modifying phrase is at the end of the sentence. The phrase a horror film about a flesh-eating
virus does not describe Eli Roth. Rather, it describes the film Cabin Fever. The modifier needs to be moved next to the
noun it modifies (Cabin Fever).

RIGHT A horror film about a flesh-eating virus, Cabin Fever was the first feature film directed by Eli Roth.

  OR: Cabin Fever, a horror film about a flesh-eating virus, was the first feature film directed by Eli Roth.

(2) In sentences that begin with a phrase, ask yourself who or what the phrase is referring to. The identity of that person or thing
MUST IMMEDIATELY FOLLOW the phrase, right after the comma.

WRONG An exceptionally gifted tennis player, Rafael Nadal’s loyal fans travel all over the world just to watch him play.

Who is an exceptionally gifted tennis player? Rafael Nadal, not his fans.

RIGHT An exceptionally gifted tennis player, Rafael Nadal has many loyal fans who travel all over the world just to watch him play.

WRONGRacing at speeds between 130 and 160 miles per hour, large crowds gathered to watch the hydroplanes.

What was racing at speeds at high speeds? The hydroplanes were racing.

RIGHT Racing at speeds between 130 and 160 miles per hour, the hydroplanes attracted large crowds.

ADJECTIVES AND ADVERBS


Adverbs modify verbs. Adjectives should not be used to modify verbs.

WRONG Judges often impose harsh sentences on defendants found guilty of treating animals cruel.

Cruel is an adjective. People can be cruel dog owners, but they do not treat a dog  cruel. How do they treat the
dog? Cruelly.

RIGHT Judges often impose harsh sentences on defendants found guilty of treating animals cruelly.
IDIOMS
Idioms are groups of words that form expressions. Expressions require exact wording. For example, one thing is  different
from another; it is not different than another. The following expressions have been tested before:
capable of
cause damage to
credited with
encouraged by
expel into
mistrust of
ought to have
renowned for [painting]
succeeded in
subject to approval
tendency to [fail]
as much as
as [happy] as
so [disappointing] that
known as
the same as
both [this] and [that]
between [this] and [that]
from [X] to [Y]
not only . . . but also
just as . . . so

NOUN-NOUN AGREEMENT
Match plural nouns with other plural nouns, and singular nouns with other singular nouns, when logic requires that the nouns
match in number.

WRON Best friends since third grade, Thelma and Louise are moving to Orlando to become a character performer at Walt Disney
G World.

Thelma and Louise cannot be a single character performer. They want to be character performers.

RIGHT Best friends since third grade, Thelma and Louise are moving to Orlando to become character performers at Walt Disney
World.

WRON Eli Whitney invented the cotton gin, machines that quickly and easily separate cottonseeds from cotton fiber.
G

The noun cotton gin is singular. The cotton gin should not be described as machines. The cotton gin is a machine.

RIGHT Eli Whitney invented the cotton gin, a machine that quickly and easily separates cottonseeds from cotton fiber.

COMMA SPLICES
Two complete sentences cannot be connected by a comma alone.

WRON Jacob Lawrence was one of the most important artists of the twentieth century, he is best known for his series of paintings
G depicting important moments in African-American history.

RIGHT Jacob Lawrence, one of the most important artists of the twentieth century, is best known for his series of paintings
depicting important moments in African-American history.

You might also like